You are on page 1of 62

ISTE

ELE¢TR
Eff&¥,##H 3.fa.-!ft.3!:EBi`i€,:ii3.`:i:5-±;;.:-<3St.`€.3:.E:`S{3:i``:.Sa::`;::.;€!.:;::i`!{a:;i;:3
i,##

a EXERCISES NO. 1 : AIgebra - 1


• EXERCISES NO. 2: AIgebra - 2
I

a EXERCISES NO. 3: Trigonometry


® EXERCISES NO. 4: Plane and Solic#
Geometry
® .EXEFtcISES NO. 5:
Analytic Geometry - 1
• EXERCISES NO. 6: Analytic Geometry - 2
a EXERcfsES NO, 7: Differential Calculus
® EXERclisES NO. S: Integral Calculus

RIPHT Review Center= REE Reviewer in M4THERATICS


Page 1
15. Find the 12th term of the expansion (x -y)15.
A. -1365x4ytl C. _|275x4yll
a. -1465X3y`2 D. -||65x3yl2

16. Find the middle term of the expansion of (x2 -


5)8`
A. 46940x6 C. 44736x8
a. 45827Xe D. 43750X8

1. Three greater than eight times a certain 17. Find the coefficient of the 6th term of the
number is 35. Determine the number. expansion (x _ 3y)12
A. 4 a. 5 C. 6 D. 7 A. -192456 C. -174416
a, ~183235 D. -165326
2. Findxjf6x-2=2x+6.
A.10 a. 5 C. 2 D. 4 18. Vvhat is the sum Of the coefficients in the
expansion of (a - b + c)8?
3. Solveforx: V(20-x)=x A- 1 a. 2 C. 3 D. 4
A. 4,-5 C. -4,5
8. ~4,-5 DI Noneofthese 19. What is the sum of the coefficients for the
expansion of ( 4x - 3)9?
4. What is/are the value of x in the expression x2 A. 16842 C. 13824
+ x -12 = 0? a. 14482 D. 19684

a-I,?26 8. =8;2 20. Find the sum of the exponents of the


expansion (x + y) raised to 10.
5. |f42X+1 = |o24, findthevalueof4X. A. 45 8. 90 C.110 D. 55
A.16 a.14, C.18 D. 20
21. Determine the sum of the exponents of the
6. Solveforxiflog381X= 16. expansion (2x + 1 )7.
A. 4 8. 5 C. 3 D. 6 A. 56 a. 28 C, 42 D. 21

7. If log of 2 to the base of 2 plus log of x to the 22. |f x4 - 2x3 - 3x2 - 4x - 8 is divided by (x - 2),
base Of 2 is equal to 2, find the value of x. the remainder is
LJ~
A. 2 a. 4 C. 3 D. 5 A. 20 a. 28 C. -20 D. -28

8. Intheequationx2+ llx+28 =0. onerootis3 23. Given f{x) = (x+4)(x - 3) + 4] when f(x) is
more than the other root. What are the roots divided by (x - K), the remainder is - K. Find
of the equation? . .the value.OfK
A. -7,-4 C. 8,5 A. 2 8. 4 C. 6 D. 8
a. -6,-3 D. 9,6
24. When ax3 + 2x2 -18x + 7 is divided by (x+1 )
9. Determine the value of k so that the sum and the remainder is -15. Find the value Of a.
the product of the rocits are equal from the A. 42 a. 45 C. 48 D. 50
given equation 7x2 + (2k - 1 )x -3k + 2 = 0.
A. -2 a. -1 C. 1 D. 2 25. Write a cubic equation whose roots are ( -1,
2,4)
10. Find the value of k from the given quadratic A.x3
equlatic>n 2x2 - kx + 6 = 0 if the sum of the
roots is equal to 4.
A. 3 a. 4 C. 6 D. 8

11. Find the value of in that will make 4x2 -4mx + 26. The radius of curvature of a given curve
4m +5 is a perfect square trinomial. varies directly with x and inversely with the
A. 3 a. -2 C. 4 D. 5 square of y. Vvhen x = 2, y = 3, the radius of
curvature is 100. Find the radius of curvature
12. Determine the value of k so that 3x2 + kx + 12 when x = 4 and y = 6.
= 0 will have just one real solution. A. 50 a. 100 C. 80 D. 120
A. 9 a. 12 C. 6 D. 16
27. The resistance of a wire varies directly with its
13. Simplify the expression: the square root of the length and inversely its area. If a certain wire
• Cube root of 64x30. 10 in lor`g and 0.10 cm in diameter has a
A. 4x5 a. 8xt° C. 2x5 D. 2xl° resistance of 100 ohms, what will be its
resistance if it is uniformly stretched so that its
14. Find the 4th term of the binomial expansion (x length becomes 12 in. Assume diameter to be
+ y)10 constaht after it is being stretched.
A. 80 8. 100 C. 120 D, 140
3.?2gx:;: 8:?2g#y:

MPHT Review center: REE Reviewer in MATHEMATICS Page 2


28. The mean proportional between 16 and x is
4. Determine the 5th term of the sequence whose
equal to 12. Find the value of x.
Sum of n terms is given by (2n+3 _ 5).
A. 3 a. 4 C. 6 D. 9
A. 258 8. 218 C. 128 D. 158
29. The electric power which a transmission line
5. Find the 100th term Of the sequence. 1.01,
can transmit is proportional to the product of
1.00, 0.99
its design voltage and the current capacity
A. 0.04 8. 0.03 C. 0.02 D. 0`05
and inversely as the transmission distance. A
240 kv line rated at 1500 amperes can 6. In an arithmetic sequence 3, 7,11, the nth
transmit 250 MW over 100 kin. How much
term is 31. Find the value of n.
power in MW can a 500 kv line rated at 2000 A. 6 a. 8 C. 7 D. 4
amperes transmit over 300 kin?
A. 230 8. 225 C. 200 D. 270 7. \^/hat is the sum of all even integers from 10
to 500?
30. The average rate of production of PCB is 1
unit for every 2 hours work by two workers. 3:?Z£?88o C. 62'7B.065,955
How many PCBs can be produced in one
month by 60 workers working 200 hours 8. How mariy terms of the progression 3, 5, 7,
during the month? should there be so that their sum will be 2600.
A. 4,000 C. 6,000 A. 60 8. 50 C. 52 D. 55
a- 5,000 D. 3,000
9. Four positive integers form an arithmetic
31. The value of pi (TT) is a number. progression. If the product of the let and the
A. Rational C. Irrational last term is 70 and the 2nd ancl the third term is
a. Prime D. Composite 88, find the lst term.
A. 5 8.. 3 C.14 D. 8
32. property states that if a < b and b <
c, then a < c. 10. The 3rd form Of a Geometric Progression is 20
A. Reflexive C. Symmetric and the 6th term is 160, find the lst term.
8. Transitive D. Identity A. 5 a. 11 C. 22 D. 4

33. If the discriminant of the quadra€ic equation is 11. A piece of paper is 0,05 inches thick. Each
less than zero, the rocks are said to be time the paper is folded into half, the
A. Real, Equal thickness is doubled. If the paper was folded
a. Real, Unequal, Rational 12 times, how thick in feet the folded paper
C, Real, Unequal, Irrational be?
D. Imaginary A. 102.4in
a. 51.2in
. _.~34. !f .b2 .~.4_ac is greater than 0 and the quadratic
ec|uation is NOT a perfect square, then the 12. Determine x, so that x, 2x + 7, 10x -7 will
discriminant is form a geometric pr.ogression.
A. Real, Equal A. 7,-5i6 c. 7.+n2
a. Real, Unequal, Rational 8. 7.-14/5 D. 7,-7/6
• C. Real, Unequal, Irrational

D. Imaginary 13. Find the sum of the first 10 terms of the


arithmetic progression 1, 3, 9 ,...
35. What do you call the first and fourth terms in A. 29524 C. 11820
the proportion Of four quantities? 8. 31760 D. 11342
A. Numerators C. Means
8. Extremes D. Denominators 14` Find the ratio of an infinite geometric series if
the sum is 2 and the fiist term is %.
A. 1/3 8. I/2 C. % D. %

15. What is the llth term Of the harmonic


progression if the first and the third terms are
1. Find the missing term: 1, 2, 6, 24, _ , 1/2 and 1/6 respectively?
720. A 1/20 a. % C. 1/12 D. 1/22
A. 250 8. 180 C. 120 D. 105
16. Find the 8'n term: 0.5, 0.167, 0.1 ,...
2. Evaluate the sum of the first n odd numbers: A. 0.033 C. 0.025
1 +3+5+ . . . i.an. a. 0.205 D. 0.412
A. n 8` 2n C. n2 D. n(n+1)
17. The sum of the ages of Peter. and Paul is 21.
3. Find the `sum of the first 5 terms in the Peter will be twice as old as Paul 3 years from
Sequence whose general term is (2n-1). nciw. What is the present age Of Peter? .
A. 16 a. 8 C. 31 D. 15 A. 8 a. 6 C. 18 D. 15

MPHT Review Center: REE Reviewer in MATHEMATICS Page 3


18. Ana is 5 years older than Beth. In 5 years the
distance. What is the speed of the boat in
product of their ages will be 1.5 times the calm water?
product of their present ages. How old is Beth
now?
A. 27 a. 20 C. 25 D. 18
3: ;33395m:£h 8: £€§:83#
28. Bing and Ryan can jog around a circular park
1 9. Maria js 36 years old. Maria was twice as old
in 8 and 12 minutes, respectively. If they start
as Arian was when Maria was as old as Arian
at the same instant from the same place, in
is now. How old is Arian?
how many minutes will they pass each other if
A. 20 8. 18 C. 24 D. 16
they jog around the track in the same
direction?
20. For a particular experiment, you need 5 liters
A.13minutes C. 15minutes
of 100/o solution. You find 7% and 12%
a. 24minutes D. 8minutes
solution on the shelf. How much of the 7%
solution you mix with the appropriate amount 29. How many minutes after 8:00 PM will the
of the 12% solution to get 5 liters of 10%
minute hand of the clock overtakes the hour
solution?
hand?
A.1.5 a. 2.5 C` 2 D. 3
A- 40/12 C. 437/11
8. 4211/12 D. 35/11
21. Ten liters of 250/o salt solution and 15 liters of
35% salt solution are poured into a drum 30. What time after Midnight will the hands of the
originally containing 30 liters Of 10% salt
clock be perpendicular with each other for the
solution. \/Vhat is the percent concentration in
1 §t time?
the mixture?
A. 10% C. 30%
a. 20%
3: 12.:8;3::: 812:]8:±f::
D. 40%

22. One pipe can fill a tank in 6 hours and another


pipe can fill the same tank in 3 hours. A drain
pipe can empty the tank in 24 hours` Vvith all
three pipes open, how long will it take to fill
1. Sin(B-A)isequaltowhenB=270°andAis
the tank?
an acute angle.
A. 5.16 C. 2.18
A` -c/OSA C. -sinA
8, 3.14 D. 1.48
a. GoSA D. sinA
23. Lorna can finish a job in 5 hours. Fe can do it
2. If sin A = 4/5, A in quadrant 11, sin a = 7/25. a
in 4 hours. If Loma worked for 2 hours and
•then Fe was asked to help her finish it, how in quadrant I, find the sin (A+B)
A. 3/5 a. 3/4 C. 4/5 D. 2/5
_. long Will Fe have tQ work with Lorna to finish
the job? 3. IftanA=1/3andcotB=2,tan(A-B)is
A. 2/5 hours C. 28hours A.11# B.-1fir C. -llff D.17
a. 4/3hours D. 1.923hours
4. Given that cos e = -12/ 13 and the e is in the
24. A man can do a job three times as fast as a 3rd quadrant, find the value of sin ©.
boy. Working together it would take them 6 A. -5/13 a.12/13 C. 5/13 D. -12/13
hours to do the same job. How long will it take
the man to do the job alone? 5. Find the values of x, where 0° £ x s 360°,
A. 9hours C. 7hours such that 2 sin2x + 5 cos x + 1 =0.
8. 8hours D.10hours A.12C}°or240° C. |006or24o°
a. 115°or320° D. |36°or31o°
25. A job could be done by twelve workers in 13
days. Five workers started the job and after 4 6. If tan 4A = cot 6A, then what is the value of
days, 3 more men were added. Find the total angle A?
number of days it took them to finish the job. A. 9o B.-loo C. |2a D. |4a
A. 21 a. 12 C. 17 D. 15
7. A road makes an angle 4,8° with the
26. It takes an airplane one hour and forty-five horizontal. How far must an automobile go up
minutes to travel 500 miles against the wind the track for it to gain 30 in in altitude?
and covers the same distance in one hour and A. 458.64m C. 285.68m
fifteen minutes with the wind. What is the a. 358.52m D. 341.56m
speed of the airplane?
8. A transmitter is located on top Of a mountain,
3: 3f3:88# 3: !88:;8# which is 3 kin high. What is the furthest
distance on the surface of the Earth that can
27. A speecl boat can make a trip Of 100 miles in
be seen from the top of the mountain? Take
one hour and 30 minutes if it travels
the radius of the Earth to be 6400 kin.
upstream. If it travels downstream, it will take
A. 205km C` 225km
one hour and 15 minutes to travel the same
a.152km D.196km

MPHT Review Center: REE Reviewer in MATHEMATICS Page 4


9. Two straight roads intersect to form an angle 20. If the angles of a triangle are 2x, x + 15, and
of 75°. Find the shortest distance from one 2x + 15, find the smallest angle in mils.
road to a gas station on the other road 1 kin A. 600mils C. 900mils
from the junction. 8. 800mils D. 700mils
A. 3.732km C. 4.365km
a. 5.325km D. 2.856km 21. A spherical triangle ABC has an angle C = 90
degrees and the sides a = 50 degrees and c =
10. Find the height of a tree if the angle of 80 degrees. Find the value of side b.
elevation of its top changes from 2o° to 4o° as A. 5|°4' C. 74°2o'
the observer advances 23 meters toward the 8. 77°52' D. 68°36'
base.
A.13.78meters C.14.78 meters 22. If Greenwich Mean Time (GMT) is 7A.M.
8. 16.78meters D. 15.78meters Vvhat is the time in a place located at 135° E
longitude?
11. Solve c of an oblique triangle ABC if a = 25, b A. 3P-M-
= 16 and C = 94.|o. 8. 4P-M.
A. 29 8. 31 C. 33 D. 35
23. If Greenwich Mean Time is 9 A.M. What is the
12. Find the height of the lamp post if the angle of time in a place 45° W Of longitude?
elevation Of its top changes from 26° to 62° as A. 7A.M. C. 6A.M.
the observer 1.8 in tall advances 32 in toward a. 4A.M` D. 2A.M.
the base.
A. 26.87m 24. An isc>sceles spherl.Gal triangle lias an angle A
a. 23.57m = 8 = 54° and side b = 82°. Find the measure
of the third angle.
13. Two towers are 60 in apart. From the top of A. 156°24'15" C. 155°35'43"
the shorter tower, the angle of elevation of the a. 158°18'43" D. 148°28'16"
top of the taller tower is 40°. How high is the
taller tower if the height of the smaller tower is 25. Considering the Earth as a sphere of radius
40m? 6400 kin. Find the area Of a spherical triangle
A. 75m C. 86m on the surfece of the Earth whose angles are
a.100m D. 90m
5o°,89o and |2oo.
14. A pole tilts tc)ward the sun at an angle 1o°
from the vertical casts a shac]ow 9 in long. If 3: 82:€Z3:8§2 g: ££:2Sg;Z€3
the angle of elevation from the tip of the
Shadow to the top Of the pole is 43°, how tall 26. Which of the following is true?
--- Js the-pole?- A. sin(-e)=sino
A. 10.2meters C. 11.3meters
8. tan(-e)=tane
8.13.7 meters D. 12.6meters C. cos (- 8) = cos 0
D. csc(-©)=cscB
15. Simplify the expression 4 cos y sin y ( 1 -2
sin2y) 27. In what quadrants do the secant and cosecant
A. Sec2y C. Tan2y of an angle plotted on a Cartesian coordinates
a. Cos2y D. Sin4y have the same sign?
A-I,1]1 a.I,11 C, ll,IV D.11,111

16. If cos © = +3 / 2, find 1 -tan2©. .


28. Of what quadrant of A, if see A is positive and
A. -1 a. -1/2 C. 2/3 D. 2 csc A is negative?
A. '11 8- lv C. I D. 11
17. The angle of elevation of the top of a light
house from a boat 50 in from it, is the 29. The north - south location of a point on the
complement of the angle of elevation Of the Earth's surface measured as an angle in
same light house when the boat is 110 in from degrees from the equator
it. Find the height of the lighthouse` A. Meridian C` Latitude
A. 56.44m C. 74.16m 8. Longitude D. Equator
a. 67.86m D. 45.67m
30. \/\/hat i§ the angle which line Of sight to the
18. If the complement of an angle theta is 2/5 of object makes the horizontal is below the eye
its supplement, then theta is of the observer?
A. 45° 8. 75° c. Goo D. 3oo A. Angleof depression C. Bearing
a. Angle of elevation D. Acute
19. A certain angle has an explement 5 times the
supplement, find the angle. 31. What do you call an angle whose terminal
A. a;rf .ff C. 1350 side cc)incides with an axis?
a. 1080 D. 58.50 A. Reflex angle C. Rightangle
8. Quadrantal D. Co-terminal

MPHT Review ce'nter: REE Reviewer in MATHEMATICS Page 5


32. Sin 2e is equal to
4. How many sides are there in a regular
A. 2sinecos© C. sinecose triacontagon?
a. %sin.e D.1-sin2© A. 20 a. 30 C. 12 D. 1000
33. An angle more .than TT radian but not less than
5. The sum of the exte+ior angles of a polygon is
2TT radjans is
the sum of its interior angles. How many sides
A. Straightangle C. Reflexangle
does the polygon have?
a. Obtuseangle D. Rightangle
A. 3 a. 5 C. 4 D. 6
34. Which of the following is NOT true about
6. How many sides has a polygon if twice the
spherical triangles?
sum of its interior angles equals thrice the
A. The sum of the sides is less than 3600 sum of its exterior angles?
a. The sLlm of three angles is |80°
A. 3 8. 4 C. 5 D. 6'
C. The sum of two sides is greater than the
third side 7. How many diagonals are there in a
D. If two sides are equal. the corresponding dodecagon?
angles opposite are equal. A. 27 8. 20 C.12 D. 54

35. 50 gradients js 8. Compute for the niimber of diagonals Of an


A. 45 degrees c. 1/2 revolution jcosagon.
8` 90degrees D. 16redians A. 20 a.170 C. 85 D. 200
36.A?43doeggfees!Sequjva!entt°cTT4¥`t'S 9. A regular hexagon is inscribed in a circle
whose radius is 10. Find the area of the
8. 2084 D. 2800 hexagon.
A. 307.12 C. 259.81
37. Two angles are said to form a ~ if they are 8, 105.18
adjacent angles and whose nan-common D. 315.72
sides are opposite rays, 10. Find the area Of a regular octagon whose
`A. Vertical angles
sides measure 5 cm.
a. Linearpair A.105.61 cm2 C. 135.67cm2
C. Complementary pair 8.148.83cm2 D.12o.71 cm2
D. F`eference angles
11. The apothem Of a regular undecagon is 8.
38. Centroid is the intersection of the of the Determine its area.
triangle. A. 206.71 C. 261.07
A. Sides 8. 170.62 D. 127.60
a. .Medians
C. Altitudes
--~---b.-I Angle bisectors 12. Find the area of a pentagram inscribed in. a
circle of radius 6 cm.
A. 27.823 sq. cm
39. It is a measurement from the north or south, a. 35.173sq.cm
clockwise or counterclockwise C. 40.428 sq. cm
A. Equator D. 54.176sq. cm
8. Bearing
C. Intemational Date Line 13. Find the area of a trapezoid whose median is
D. Prime Meridian 54 cm and whose altitude is 8.
A. 192 8. 432 C. 342 D. 243
40. In the curve y = tan 3x, what is its period?
A. 3TT a. TT/3 C. 2"/3 D. TT 14. Find the area of a circle inscribed in a
rhombus whose perimeter is 56 1/3 in ancl
whose longer diagonal is 26 in.

3: £3.:g.::. jn g: Zgz:€9Sgd:Tn
1. What type of polygon has all of its interior
angles less than 180 degrees? 15. The lerigth of the side of a rhc)mbus is 10 cm.
A. concave C. irregular If its shorter diagonal is of length 12 cm. What
a. convex D. regular is the area Of the rhombus?

2. Given a polygon, the sum of its exterior g: S8:a::# 3: §5:a:::


angles is,
A. oo a. |8oo c. 36oo D` goo 16. A rhombus is formed by two radii and two
chords of a circle of radius 6 in. What is the
3. Polygons are namecl according to their area of the rhombus?
number of A.15.5sq.in
A. diagonals C. .edges 8. 52.3sq.in
a. extenor angles D. faces

MPHT Review Center: REF Reviewer in MATHEMATICS


17. Find the fourth side of a quadrilateral 27. A spherical wooden ball 15 cm. jn diameter
inscribed in a circle having one of its sides sinks to a depth of 12 cm. in a certain liquid.
equal to 13 in. as its diameter, and the other Find the area exposed above the liquid.
two sides adjacent to the diameter are 5 in.
and 8 in., respectively. g g88i g. f53:
A. 4.271 in C. 5.781m
a. 3.812m D. 6.382m 28. What is the area of a lune whose angle is 85.
on a sphere of radius 30 cm.
18. The sides of a cyclic quadrilateral are a = 3
cm, b=3 cm, c=4 cm anc! d=4 cm. Find the 9. i:833:35 ::: g. i:gig:8g ::2
radius of the circle that can be inscribed in it.
A.1.71cm C. 3.13cm 29. Find the area of a spherical triangle ABC,
8. 2.71 cm D. 4.71cm A=| |5°, B=70°,C=92° in a sphere of radius 12
Cm.
19. A circle having a radius Of 6.5 cm. is A. 143.44cm2 C. 343.65cm2
circumscribing a triangle having an area of 30 a. 243,78 cm2 D.117.86cm2
sq. cm. If one side of the triangle is 13 cm.,
find the shortest side of the triangle. 30. Find the surface area of regular icosahedrons
A.12cm a.10cm C. 5cm D. 8cm when each edge is of length 5`
A. 216.5 C. 126.6
20. An engineer places his transit along the line 8. 261.5 D. 162.5
tangent to the circle at point A such that
PA=200 in. He locates another point a on the
circle and finds PB=80 in. If a third portion C,
on the circle lies alorig PB, now far from point
a will it be?
A. 500m C. 480m 1. On what quadrant does the coordinate (-2, 6)
a. 420m D. 420m lies?
A. I 8. 11 C. Ill D. 'V
21„ The perimeter of a sector is 9 units and its
radius is 3 units` What is the area of the 2. Given the points (3,7) and (4,-7). Solve the
sector? distance between them.
A. 5 8. 6.5 C. 4.5 D. 7
A. 15.65 C. 16.65
a. 17.65 D. 14.65
.22. A cylinder is circumscribed about a right prism
having a square base one meter on an edge. 3. The distance between (2,-9) and (x, 3) is 15.
The volume of the cylinde-r is 6.283 cu. in. Find the x.
Compute its altitude. A. 10 8. 11 C. 12 D. 13
_..A. 3 8. 4 _C. 5 . D. 6
4. How far is tr`e intersection of the lines x + 2y -
23. The volume of a truncated prism with an 12 = 0 and 3x -5y + 15 = 0 from the origin?
equilateral triangle as its horizontal base is A` 10.2 a. 6.3 C. 1.25 D. 5.38
equal to 1800 cu. cm. The vertical edges at
each corners are 3, 4, and 5 cm., respectively. 5. A point P(x,2) is equidistant from the points
Find one side of the base. (-2,9) and (4,-7). The value of x is
A.10.39cm C. 32.23cm A. 11/3 a. 20/3 C. 19/3 D. 6
a.15.87cm D. 27`17cm
6. The segment from (-3, 4) to (1,-2) is extended
24. A cone is inscribed in a hemisphere of radius three times its own length. Find the terminal
r. If the cone and the hemisphere share point.
bases, find the volume of the region inside the A. (-13,-20) C. (13,-20)
hemisphere but outside the cone. a. (-13,20) D. (13,20)
A. 4iTr3/3 C, TTr2h/3
a. 2TTr3/3 D. TTr3/3 7. Determine the coordinates of the point which
is three-fifths of the way from the point (2, -5)
25. A wedge is cLit from a cylinder of radius 3 in to the point (-3, 5)
by two planes, one perpendicular fo the axis A. (-2,1) C. (1,-1)
of the cylinder and the other passing through a. (2,1) D. (-1,1)
the diameter of the section made by the first
plane and inclined to this plane at an angle of 8. The line segment connecting (x, €) and (-2, y)
45: Find the volume Of the wedge? is bisected by point (2, -1 ). Find the value of x.
A.16m3 8.18m3 C. 20m3 D. 22m3 A. 4 a. 5 C. 6 D. 7

26. If the edge Of the cube is decreased by 12%, 9. ` Locate the centroid Of the triangle ABC having
by what percent is the .surface area coordinates A(0,4), a(2,6) and C (1,4).
decreases? A. 1,2 a. 2,0 C. 1,4 D. 1.-2
A. 23% 8. 77% C. 73% D. 27%

MPHT Review center: REE Reviewer in MATHEMATICS Page 7


10. Fincl the area of triangle whose vertices are A22. What is the equation of the line bisector of the
(~3, -1), 8 (5, 3) and C ( 2, -8). acute angle formed by the intersection of the
A. 37 a. 28 C. 38 D.
17 lines 4x + 3y -24 = 0 and 5x -12y + 30 = 0?
A. 9x+33y=154
11. In a Cartesian coordinates, the coordinates of a. 33x+99y=154
a quadrilateral are (1,1), (0, 8), (4, 5), and C. gx-33y=154
(~3, 4). What is the area? D. 33x-9y=154
A. 16 a. 20 C. 18 D. 25
23. Determine the acute angle between the lines
12. Find the area of the hexagon ABCDEF formed
y - 3x = 2 and y - 4x = 9`
by joining the points A(1,4), a(0,-3), C(2,3),
D(-1,2), E(-2,-1 } and F(3,0). 3: 3:8€8:8 8: 3:8?a:a
A. 24 8. 20 C. 22 D. 15
24. The distance between points (5, 30°) and (-8,
13. Are the lines represented by the equations y = -500) is:
3x + 2 and 6x + 2y = 5 parallel? A. 9.84 C. 6.13
A. No! a. 10.14 D. 12.14
a. Yes!
C. Cannot be determined 25. Find the polar coordinates for the point whose
D. Atonlyagiven range rectangular coordinate of (€, ng).
A. (10, -233.23o)
14. \^/hat js the equation of the line that passes a. (10, 233.23o)
through (4, 0) and is parallel to the line x ~ y - I C. (10.126.|87o)
2=0? D. (10,-53.|3o)
A. x+y-2=O
8-y-x+4=0 26. Transform the equation below into Cartesian
coordinates:
15. 'What js the equation of the line that passes
3
through (-3, 5) and is parallel to the line 4x -
2y + 2 = 0? 3 + 2 cos 0
A. 4x+2y+13=O A 5#-9y2 + 12x + 9 = 0
8. 2x+4y-17=O 8 5x2 + 9y2 - 1 2x - 9 = 0
C-x-2y+15=O C
D. 2x-y+11=0 D gx:=3y2=;2X=3=8
16..Find the equation of the perpendicular
bisector of the line joining (4,a) and (6,3).
A. 4x+6y-29=0
_..I _ 8. . .4X +.6.y.I 29 .=.0
C. 4x-6y+29=O 1. What conic section is represented by the
D. 4x-6y-29=0 equation 4x2 + 3y2 -8x + 16y +19 =0
A. Circle C. Hyperbola
17. A line 4x + 2y -2 = 0 is coincident with line a. Ellipse D. Parabola
A. 4x+4y-2=O
a. 4x+3y+3=O 2. What is the nature of the curve represented
C-8x+4y-2=O by the equation x2 + 4y2 + 4xy + 2x - 10 = 0?
D. 8x+4y-4=0 A. Circle C. Hyperbola
a. Parabola D. EIlipse
18. What is the distance Of the line 4x -3y + 5 = 0
from the point (4,2)? 3. Vvhat conic section is represented by the
A. 5 a. 4 C. 2 D. 3 equation 4x2 + 8x - y2 + 4y - 1 5 = 0?
A. Parabola C. Hyperbola
19. Find the value of k if the distance from the 8. EIlipse D. Circle
point (2,1 ) to the line 5x + 12y + k = 0 is 2.
A. 5 a. 2 C. 4 D. 3 4. Determine the nature of the curve represented
by the equation x2 + y2 -8x + 16y + 81 = 0.
20. Find the distance between the lines, 3x + y - A` Circle C. Parabola
12 = 0 and 3x + y -4 = 0. a. Hyperbola D. Emptyset
A. 2.53 a. 1`14 C. 3.17 D. 1.75
5. What is the center of the curve x2 + y2 - 2x -
21. What is the equation of the line through (-3. 5) 4y -31 = 0-
which makes an angle of 45 degrees with the A. (-1,-2) C. (1,-2)
line 2x + y = 12? a. (-1,2) D. (1,2)
A. x+2y-12=O
8-x+2y-18=O 6. What is the center of the circle x2 + y2 +.12x ~
C. x+3y-12=O 8y + 16 = 0
D. x-3y+12=0 A. (6,4) C. (6,4)
a. (€,4) D-(€,4)

MPHT Review center: REE Reviewer in MATHEMATICS Page 8


7. Find the equation of the circle whose center is 19. Find the ratio of the major axis to the minor
at (3, -5) and whose radius is 4. axis of the ellipse: 9x2+4y2-72x-24y-144=0.
A. x2+y2+8x-4y+11=O A. 0.67 8. 0.33 C. 1.67 D. 1.5
a. x2+y2-6x+10y+18=O
C. x2 t y2-lox-2y + 1 = 0 20. What is the equation of an ellipse, major axis
D. x2+y2+14x+16y+117=0 horizontal, with center at (1, 2) and whose
major and minor axis are 6 and 4,
8. Find the area of the curve given the equation
x2 + y2 + 4x - 8y - 5 = 0 i:Spxe2C¥V4e}¥?i2x_8y+1=0
A. 24.8squnits C. 78.5squnits
a. 81.7squni{s D. 95.2squnits
:-:::±§#_3::-x-3_3;:+9-4=4=:8=°
9. Find the equation of the circle passing through
21. Find the eccentricity of an ellipse given the
;h.efgjTt;2(=32;`2ij;i_`}!fa=do(5i3). equation: 4x2 + 25y2 -8x - 100y -296 = 0
A. 1.3 8. 0.92 C. 2.5 D. 0.27
3. X::;::8;=::;:i8:8
D. x2+y2-8x-12y+3=0 22. The area of the ellipse 9x2 + 25y2 - 36x - 189
= 0 is equal to:
10. What is the equation of a cirole whose ends of A. 20TT 8. 15TT C. 10TT D. 25TT
diameter are (10, 2) and (6,4).
A. x2+y2-8x+2y-48=O 23. Find the latus rectum of the ellipse 9x2 + 25y2
8. x2+y2-|6x+2y+52=O - 36x - 1 89 = 0.
C. x2 + y2 + |6x-12y + 52 = 0 A. 1.8 8. 0.3 C. 2.8 D. 3.6
D. x2 + y2 + 16x-2y + 48 = 0
24. Given an ellipse x2/36 + y2/32 = 1. Determine
11. Compute the focal length and the length of the distance between foci.
latus rectum Of parabola y2 + 8x -6y A. 2 8. 3 C. 4 D. 5
A. 2,8 C. 4,16
8.16,64 D.1,4 25. Find the center of the hyperbola: 4x2 - 9y2 +
8x - 18y - 149 = 0.
12. Find the focal length Of the curve 2x2 -8x -
4y + 16 = 0. 3: {2t,'_i} g {=1:2t,j
A. 2 a. 1 C. 0.5 D. 0.25
26. Find the transverse axis of the liyperbola: 4x2
1 3. The parabola x2 + x + y + 1 = 0 opens: _ 9y2 + 8x - 18y - 149 = 0.
A. 6 a. 4 C. 12 D. 8
3: I:;ia:r!eft g: I:;hnewri3:t
27. Find the eccentricity of the curve 9x2 - 4y2 -
14. Find the equation of the parabola that passes 36x + 8 y = 4.
A. 1.8 8. 1.2 C. 1.5 D. 1.6
Ah.r°g£2th4exp_°jtn8;(+3t36±16-2.1.5).("2).
a. 2y2-6x+5y~25=O 28. What is the semi-conjugate axis of the
C. x2-8x+4y-15=O hyperbola x2/9 - y2/4 = 1 .
D. 3x2+5x-3y+10=0 A. 1 8. 2 C. 3 D. 4

15. Find the value Of the parabola whose axis is 29. What is the equation of the upward asymptote
vertical and passes through (-1, 0), (5, 0), (1. of the hyperbola (x -2)2/9 -(y + 4)2/1 6 = 1 .
8) and (4, y). A. 4xt3y-20=O
A. -5 a. 5 C. -6 D. 6 8. 4x-3y-20=O
C. 3x+4y+20=O
16. An arch 18 in high has the form of parabola D. 3x-4y-20=0
with a vertical axis. The length of a horizontal
beam placed across the arch 8 in from the 30. Find the eccentricity of a hyperbola Whose
top is 64 in. Find the width of the arch at the transverse and conjugate axes are equal in
bottom. length?
A.106m C-54m A. 1.56 8. 1.41 C. 0.76 D. 2.31
a. 74m D. 96m
31. Find the latus rectum of the curve 4x2 - 9y2 +
17. The center of the ellipse 4x2 + y2 -|6x -6y - 8x - 1 8y - 1 49 = 0.
43 = 0 is at: A. 2.18 8. 5.33 C. 0.88 D. 3.16
A. (1,1) C. {-3,4)
8. (2,3) D. (2,1)

18. Find the major axis of the ellipse x2 + 4y2 - 2x


- 8y + 1 = 0.
A. 2 a., 4 C. 10 D. 6

MPHT Review Center: REE Reviewer in MATHEMATICS Page 9


15. Find the equation of a line normal to the curve
'- I ,` `
.----.. i .. ` x2 = 1 6y at (4, 1 )

1. Evaluate: tim (1-cosx)/x2 ; x+0 3-g;iy,=-823 g: 2f;yy=-;


A. 0 a. 1/2 C. 2 D. -1/2
16. The sum Of two positive numbers is 50. What
2. Iim(tan2x-2sinx)/x3;x+0 are the numbers if their products is to be the
A. 0 a. 3 C. infinity D. DNE largest possible.
A. 24&26 C.25&25
3. |im (x2-1)/(x2+3x-4); x-1 8. 28&22 D.20&30
A. 1/5 a. 2/5 C. 3/5 D. 4/5
17. A triangle has variable sides x,y]z subject to
4. |im (3x4_2x3 + 7) /(5x3+ x-3); x~jnfinity the constraints such that the perimeter js fixed
A. 0 8. Infinity C. -1 D. DNE to 18 cm. What is the maximiim possible area
for the triangle?
5. |im(x3_x2-4)/(x2-4);x+2 A.15.59cm2 C.17.15cm2
A. 1 a. 2 C. 3 D. 4 8.18.71 cm2 D.14.03cm2

6. x2
Differentiate the equation y = ;i=T 18. Two post one is 10m high and the other 15m
x2 + 2x riigh stand 30m apart. They are to be stayed
C.2x by transmission wires attached to a single
Eiiii
X stake at ground level, the wires running to the
D.
ii-!|T top of the posts. Where the stake should be
placed to use the least amount of wire.
7. Differentiate y = excosx2 A. 12m a. 14m C. 18m D. 16m
A. ngxsinx2
8. ex(cosx2-2xsinx2) 19. A fencing is limited to 20ft length` What is the
C. excosx2-2xsinx2 maximum rectangular area that can be fenced
D. -2xexsinx in using two perpendicular comer sides of an
existing wall?
8. Differentiatey = |ogio(x2 + 1)2 A.120 a.100 C.140 D.190
A. 4x (x2+1)
8. (4xlogioe)/(x2+1) 20. A poster is to cbntain 300(cm Square) of the
C. Ioge(X)(X2+1) printed matter with margins of loom at the top
D. 2x(x2+1) and bottom and 5cm at each sides. Find the
overall dimensions if the total area of the
9. Differentiate (x2 + 2)1ca poster is minimum.
C. 2X / (x2 + 2)1/2 A. 27.76cm, 47.8cm
3:_ {X; (:2?i"2,'iz2 •-D. .(x2 +2)3ra~
8. 20.45cm, 35.6cm
C. 22.24cm, 44.5cm
10. Find the equation Of the normal line to x2 + y2 D. 25.55cm, 46.7cm
= 5 at the point (1,2)
A. 2:x-y=O 21. Water is flowing in a conical reservoir 40 ft
8. 3x+y=0 deep and 16 ft across the top at the rate Of 2
ft3/min. Find how fast the surface is rising
11. Determine the velocity when t = 4 given the when the deep is 4 ft deep.
equation D = 20t + (5 / t + 1) where D is in A. 0`85ft/min C. 0.83ft/min
meters and t is in seconds a. 0.95ft/min D. 0.66ft/min
A. 1g.8m/s C. 13.4m/s
a. 12.6m/s D.14.5m/s 22. The cost per hour of running a motor boat is
proportional to the cube of the speed. At what
12. If y = 4 cos x + sin 2x, what is the slaps of the speed will the boat rlJn against a curl.ent of 8
curve when x = 2 radians? kin/hr in order to go a given distance more
A. -2.21 C. -3.25 economically?
a. -4.94 D. 2.21
3: :8# 3: 1i#
13. Locate the points of inflection of the curve y
f(x) = x2 ex . 23. AI any distance x from the source Of light, the
A. ~2±f i3 c`-2±rfe2 intensity of illumination varies directly as the
8. 2±VZ D.2±/§ intensity of the source and inversely as the
square of x. Suppose that there is a lighi at A,
14. Given the curve y = 12 -12x + x3, determine and another at 8, the one at 8 h.aving an
its maximum, minimum and inflection point. intensity 8 times that of A. The distance AB is
A. (-2,28) , (2,-4) , &(0,12) 4 in. At what point from A on the line AB will
8. (2,-28) , (2,4) , & (0,2) the intensity Of illumination be least?
C. (-2,-28) , (-2,4) , & (2.,12) A. 2.15m C.1.50m
D. (-2,28) , (-2,4) , & (1,12) a.1.33m D.1.92m

MPHT Review Center: REF Reviewer in MATHEMATICS Page 10


24. There is a constant inflow of liquid into a
conical vessel 15ft deep and 7.5ft in diameter.
Water is rising a{ the rate of 2ft/min when the 4. Evaluate J{2Jo2y(x2 +y2)dxdy
water is 4ft deep. \/\/hat is the rate of inflow jn
A.16.5 8`15.5 C.17.5 D.14.5
ft3 per minute?

3: 2:3g #:;:i: g: 8:636ft¥,3fT+.n 5. Evaluate Jo2Jo3Joydxdydz


A.13 a.9 C.7 D.5
25. Water drains from a hemispherical basin of
diameter 20 inch at the rate Of 3 in3 per 6. The integral of34Xdx is equal to:
second. How fast is the water level falling
when the depth of water is 5 inch? 3: :::;(;:2?)++cc 8 3::;(:::i) : 8
A. 0.012732 inch/see
a. 0.021732 inch/sec 7. What is the integral of (2sec2x-sin x)dx?
C. 0.000232 inch/see A. 2 cos x + tan x + C
D. 0.000323 inch/see 8. 2 tan x + sin x + C
C. 2 sin x + cos x + C
26. A standard cell has an emf "E" of 1.2 volts. If D. 2 tan x + cos x + C
the resistance "R" of the circuit is increasing at
the rate of 0.03 ohm/see, at what rate is the
current "I" changing at the instant when the
resistance is 6 ohms? Assume Ohm's law E = 8. Evaluate JX2Jfadx
IR.
A. -0.002 amp/sec A. £(X3+3)3'2+c C. €(X3+3)-"2+c
8. 0.004 amp/sec
C. -0.001 amp/sec 8. :(X3+3)2'3+c D. 3(X3+3)3'2+c
D. 0.003 amp/see

27. A point on the curve where the second 9` Evaluate: Jy3JZFrdy


derivative of a function is equal to zero is
called: A. ±(2y3 -1)3`2(3y3 +1)+c
A. Maxima
a. Minima a. ±(2y2 -1)"2(3y. +1)+c
C. Point of Inflection
D. Point of intersection C. ±(2y+1)"2(3y2 +1)+c

D. ±(2y2+1)3'2(3y2 _1)+c
28. The point on the curve where the first
derivative of a function is zero and the
second derivative is p.osi|~ive is called:
A. Ma*iin-a lo. ln-regrate. apdx
a. Minima
C. Point of Inflection A. See(1/x)-cot2(1/x)+c
D. Point of Intersection a. tan(1/x)-x+c

29. At the minimum point the slope from the y- c. cos(1/x)+c


axis' D. Sin(1/X)-csc2(1/x)+c
A. Negative C- Positive
8. Infinity D. Zero
11. Evaluate: Jey`4y3dy
30. At the point of inflection where x=a
A. f"(a) notequal0 C-f„(a)>O A:yF±x. +c c. eN. +y3 +c
a. f"(a)=0 D-f'(a)<0 B. ey`+4y+C D. ey`+C

t22ffinthegraphsofy=
1. Evaluate the integral sin4 Ode from o to pi/2.
A.2n/3 B.37c/5 C.3Ofl6 D.7c/4

2. What is the integral of cos xdx from pi/3 to


pi/2?
A.0.134 a.0.500 C.0.707 D.0.293 A.1 a.2 . C.3 D.4

3. What is the integral of sin6 a cos4 a d® if the 13. Compute the area of the plane region
upper limit is iT/2 and the lower limit is 0. bounded by trle curve x = yz -2 .and the line y
A. 0.0184 C. 0.1398 =-X.
a.1.0483 D. 0.9237

MPHT Review Center: REE Reviewer in inATHEMATlts Page 11


A. 912 a.1715 C.12/7 D.12.80
19. Determine the coordinates of the centroid Of
the area bounded by the curves x^2 = -y + 9,
the co-ordinate axes Of the lst quadrarit.

A.28.12 a.18.02 C.21.08 D.12.80

15. Find the area of one leaf of the four-leaved


clovers Of r = 4 sin 2e, C.1.125, 3.6
D.1-5, -3.4

20. The curve has an equation y = ex. Compute


the centroid from the y-axis of the area
bounded by the curve from x = 0 to x = 1 .
A.0.335 a.0.146 C.1.899 D.0.582

21. The region in the first quadrant is bounded by


the curve y2 =4x and the lines x = 4 and the x-
A. IT B. 2TT C. 4IT D. 8TT
axis. How far is the centroid of the curve from
the x-axis?
16. Find the area inside trie cardioid r=.4(1+cose)
A.2.4 a.1.5 C.0.8 D.1.8
but outside the circle r = 4.
22. Find the moment of inertia Of the area
bounded by the curve y2 = 4x, the line x = 1,
the x-axis on the first quad.rant with respect to
x1 y-axis.
A.0.571 a.0.682 C.0.436 D.0.716

23. Find the moment of inertia of the area


bounded by the curve x2 = 4y, the line y = 1
and the y-axis on the first quadrant with
A. 22.28sq.units C.11.14sq.units respect to x-axis.
B. 44.57sq. units D. 66.71 sq. units A.6i5 F3.7i2 C.4r7 D.8r7

24. Find the moment of inertia of the area


17. The curve has an equation of x2 + y2 = 25. bounded by the curve y2 = 4x, the line y = 2,
Find the length of the arc on the first quadrant the y-axis, on the first quadrant with respect to
from x = 3 to x = 4. y-axis.
A.0.095 a.0.064 C.0.088 D.0.076

25. Given the area in the first quadrant bounded


by x2 = 8y, the line x = 4 and the x-axis. What
is the volume generated by revolving this area
about the y-axis?
A. 78.987 cu.units
a. 50.265 cu.units
D. 82.285 cu.units

26. Find the volume by revolving the hyperbola xy


18. Find the length of the curve r = 4sin e from e = = 6 from x = 2 to x = 4 about the x-axis.
0 to e = goo and also the total length of curve. A. 28.27 cu.units
8. 25.53 cu.units
3-3:; 8: 3: #gIT C. 23.23 cu.units
D. 30.43 cu.units

MPHT Review Center: REF Reviewer in MATHEMATICS


Page 12
27. The area bounded by the graphs of y = 2x + 3
and y = x2 revolves about the x-axis.
Determine the volume generated. 6. Obtain the differential equation of the family of
A.228 8.329 C.255 D.375 straight lines with slope and y-intercept equal.
A. ydy+(x+1)dx=O
28. Given the area in the first quadrant bounded a. ydx+(x+1)dy=O
by x2 = 8y, the line y - 2 and the y-axis. What C. ydy-(x+1)dx=O
js the volume generated when this area is D. ydx-(x+1)dy=0
revolved about the line y -2 = 0?
A. 53,31 cu.units 7. Write a differential eqLiatjon for the family of
8. 45.87 cu.units circles with center on the x-axis.
C. 26.81 cu.units A-y"+y'+1=o
D. 33.98 cu.units a. yy„ + (y')^2 + 1 = 0
C- y„ + (y,)^2 + 1 = o
29. Determine the volume generated by rotating D. yy"-(y')^2=0
the curve 9x^2 + 4y^2 = 36 about the line 3x +
4y = 20. 8. Obtain a genera' solution to the following
A.52TT2 B.96TT2 C.26Tr2 D.48TT2
differential equation:
(1 + y2)dx + (1 + x2)dy = 0
30. The curve has an equation of x2 + y2 = 25. A. tanx+tany=C
The area on the first quadrant of the curve is a. tan-'x-tan-1y+C
revolved about the line x = 10. Determine the C. tanx~tany=C
volLlme of the solid of revolution. D. tan-1x+ tan-1y= C

9. Obtain a particular solution for the following


3-37! i8 g: 88;:i2
clifferential equation, given x = 2 and y = 3.
31` Calculafe the work done in pumping out the 2xyy, - 1 + y2
water filling a hemispherical reservoir 3 in A. y=sqrt(5x-1)
deep- 8. y=sqrt(2x+1)
A.550kJ B.325kJ C.450kJ D.624kJ C. y=sqrt(x-1)
D. y=sqrt(3x+2)

10. Solve for the general Solution of the following


differchtial equation:
3(3x2 + yz)dx -2xydy = 0
1., Determine the order Of the following A. x^3 = c(x^2 +9y^2)
a. x^2 = c(9x^2 i-y^2)
differential equation:
C. x^3 = c(9x^2 + y^2)
D. x^2 = c(x^2 + 9y^2)
(5¥)3+3y(¥)7+y3(£2=5x
A.1 8. 2 C. 3 D. 4 11. Which of the following differential equations is
ENACT?
2. Determine the degree of the following A. {x+.y)dx+(x-y)dy=O
differential equation: a. (x+2y)dx+(x-y)dy=O
C. (x+y)dx+(2x-y)dy=O
D. All of the choices are correct
(¥)3+3y(¥)7+y3(¥)2=5x
A.18. 2 C. 3 D. 4 12. Solve for the general Solution of the following
DE: 2ry dx + (1 + x2)dy = o
3. What is the order and degree of the following
clifferential equation? 3: X;!:!:8 8: £;2:;:8
yfy")4 + 2x(y')5 = 8xy2
A.1,5 8.3,4 C.4,2 D.2,4 13. Which of the following differential equations is
considered nan-linear?
4. Eliminate the arbitrary constant in the A. (x^5+3y)dx-xdy=O
following equation: a. yy"-2ycclsx=sinx
x3 - 3x2y = c C. y"-2cosxy' =sinx
A. (x-2y)dx-xdy=O D- y'-(2/x)y=x
8. (x+2y)dx-xdy=O
C. (x-2y)dx+xdy=O 14. Determine the integrating factor for the
D. (x+2y)dx+xdy=0
Lo.IIogingdig.en:T3tia'eq8.atj:3nky'-D3`y=3€
5. Eliminate the arbitrary constants Ci and C2 in
the following equation: 15. Solve for the particular s6Iution of the`,
y = cfe-2X + c2e3X differential equation in the previous problem
A. y' -V -e;ry =0 given that y = 6 when x = 0`
a. y„-2y'-6y=O A\ y=&E5RA_2 C. y=8e-3x_2
C. 2y„-y'-6y =0 a. y=4e3x+2 D. y=4e-3X+2
D. 2y„-y'-6y=0

MPHT Review Center: REE Reviewer in MATHEMATICS


Page 13
16. Solve for the general solution of the following
dl.fferential equation: y` + ry = xy2
A. y=Ce(X^2)J2+ 1
8. y=Ce-(X^2)J2+ 1
C. y-1= Ce(X^2)J2 + 1 For problems # 1, 2, 3, 4 & 5.
D. y-1=Ce-(x^2)J2 + 1 Given: Two Vectors
A=(1,2) 8=(3,4)
17. Solve for the general solution of the fowowing 1. Whatisthe magnjtudeofA?
differential equation: y" - 3y` + 4y = 0
3. 2.333 g: 8.8£Z
A.y=el.sxccLcOsJ=7=x+c2sinJ5%x}
8. y = e-1.5X(c]cos¥x+c2sin¥X) 2. What is the magnitude ofB?
A. 3 8. 5 C. 7 D.9
c_. y = el`5:_(cLcosf ix + c2stinjix)
D. y = e-1..5X(c[ cosV7x + c2 sinv7x) 3. Determine A.B (dot product).
A. 18 a. 15 C. 20 D. 11
18, Determine a third -order differential equation
with real numbers as coefficients if two 4` Determine A X B (cross product).
solutions are known to be e-2X and sin 3x.
A. y"+2y„+9y'+18y=o 3 (8:5:-78') 8. (8:8;:3)
a-y„'t3y"+4y,+5y=o
C. y„'+4y„+3y,+5y=o 5. Determine the angle formed byA and B.
D. y"+9y„+2y,+18y=O
3: i8:28;: 8: :8-193:
19. Solve for y: (D3 -3D2 + 3D - 1 )y = o
A. y=e-x(ci +xc2+x2c3) 6. Suppose P is the point (-1, 8) and Q is the
a. y = ex(ci + xc2 + x2c3)
point (3, 2). Find the vector A having P¢ as a
c. y = ex(ci + c2 + c3) representation`
D. y = e-X(ci + c2 + c3)
g. ::: f: 8: :2?.-;:>
20. A bacteria culture is known to grow at a rate
proportional to the amount present. After one 7. Given A = 3i + i and a = -2i + 4j, find the unit
hour, 1000 strands of the bacteria are vector having the same direction as A - 8.
observed in the culture; and after four hours,
3000 strands. Find the approximate number of
Strands Of the bacteria originally in the culture. i.
8. ±i-=j
=i+±j c-±i-±j
A. 496 a. 964 C. 694 D. 946
D. ±i+±j
8. Given A = 3i + 2j and 8 = 2i + kj, where k is a
21. A metal bar at a temperature Of loco F is
scalar, find (a) k such that A and a are
placed in a room at a constanttemperature Of- - orthogonal; (b) k such that A and a are
OOF. |f after 20 minutes the temperature of the
parallel.
bar is 50° F, find the time it will take the bar to
reach a temperature of 25° F 3: :3:€;io 8: :8::;3
3. 33-8:i: g: f8:8::: 9. Given the vectors
A=-5i+j B=4i+2j
22. A body of mass 5 slugs is droppecl from a Find: (a) the scalar projection of 8 onto A; (b)
height of 100 ft with zero velocity. Assuming the vector projection of a onto A.
no air resistance, find the time required to
reach the ground.
A. (a)-#, (b,=i-±j
A. 2.5secs C. 3.Oses a. (a)-a,(b,=i-=j
a. 2.Osecs D.1`5secs
c. (a,-#(b,=i-£j
23. A tank initially holds 100 gal of a brine solution D. (a)-±, (b,=i-±j
containing 1 lb c]f salt. At t = 0 another brine
solution containing 1 lb of salt per gallon is 10. Find ttle area of a parallelogram with two
poured into the tank at the rate of 3 gal/min, sides identified by vectors from the origin to
while the well-stimed mixture leaves the tank the points (3.4) and (8,0).
at the same rate. Fincl the time at which the A, 32 8. 24 C. 12 D. 36
mixture in the tank contains 2 Ib Of salt.
A. 0.432min C. 0.338min 11. Find the undirected distance between the
a. 0.823min D. 0.243min
pciint§ P(~3, 4, -1) and Q(2, 5, +).
24. Find the orthogonal trajectories of the family
A. vacs 8. ise5 c. 7 .D. /g§9
of curves y = cx2
12. Given A = <5, -2, 6> and a = <8, -5, 4>, find
A. 0.5x^2+y^2=k
A+a.
a. x^2+y^2=k
A. =16,4,-2=
C. x^2+0`5y^2=k
a. =15,-5,-1=
D-x^2-y^2=k

MPHT Review Center: REF Reviewer in MATHEMATICS


13. Given A = <5, -2t 6> and a = <8, -5, 4>, find
26. The scalar projection of the vector a onto the
A-a.
vector A is
A- <-5, 5' 8=
A. 3.73 a.4.15 C.5.46 D.6.78
a. =-4,4.9=
27. The vector projection of the vector C onto the
14. Given A = <+, 7, -2>, find 3A and -5A.
vector a is
A. <-12, 21. -6>, <20, -35,10>
A. -1.143i+4.573j+2.287k
8. >' =19' -35,10=
8. -2.182i-1.309j+ 1.528k
<18, -35,10=
C. 6.080i-1.765j-0.196k
<17, -35.10=
D` 1.961i+1.177j-1.372k
15. Given R(2, -1, 3) and S(3, 4, 6), find the unit
28. Find the area of the triangle whose vertices
vector having the same direction as V{k§}. are A (3,1,2) , a (4, -2,1) and C (1,1,3)
A. ±i+±j+ ±k A. 3.39 8.4.30 C.5,38 D.6.74
8. J=i+J=j+ J=k
For problems # 29 - 32.
c, ±i+±j+ ±k Given the position vectors A and a of a
rectangular coordinate system.
D. ±i+7=j+ =k A(2,4,3) a(1,-5,2)
29. Compute the resultant in forms of unit vectors
16. Find the distance between the two points (3, 4 i, j, and k.
-6) and (4, -7, 8) using vectors.
A. 17.83 8. 18.37 C. 13.78 D. 15.64 3: 2i=J;jT+5§k g: g;=gj=2{
17. Points C (5, 7, z) and D (4,1, 6) are 7.28 cm 30. What is the magnitude of the resultant vector?
apart. \/\/hat is the value of z? A. 5.92 a. 4.12 C. 7.07 D. 6.71
A. 3cm B.2cm C.4cm D.1cm
31. Tlie scalar projection of vector A onto vector a
18. Determine the scalar product at (1, 2, 3) is
A. 144 8. 138 C. 132 D. 126 A. 2.19 8.-2.19 C.2.22 D.-2.22

19. What is the cross product A x a of the vectors, 32. Vvhat is the vector projection of vector a onto
A = i + 4j + 6k and a = 2i + 3j + 5k? vedior A:R
A. <-.828i -1.655j -1.241k>
3: i++kk g: 2!=7j.=gk a. <4.27j 0.19j -2.60k>
C. <4.20i -0.18j -2.56k>
For problems # 20 - 27. D. <-4.20i 0.18j 2.56k>
Assume the three force vectors intersect at a
singlepoint L..I 33. V\mat..is the. angle-between..t\ro vectorsr.A and
A = i + 3j + 4k, a = - i + 4j + 2k, C = 2i + 7j - k 8 if A = 24i -8j + 6k and a = 4i + 12j + 6k?
A. 76°26' C. 87°|9'
20. What is the magnitude of the resultant force 8. 84°20' D. 92°6'
vector R?
A. 15.0 8` 13.2 C. 14.7 D. 16.2 34. Compute the value of b if A and a are
perpendicular. A = 2i + bj + k, 8 = 4i -2j -2k
21. Find the magnitude of displacement from A. 3 8. 2 C.1 D. 4
vector A to vector a.
A. 2 a. 3 C. 4 D. 5 Given a vector V = (x2y)i -(xy)j + (xyz)k.
35. Determine the divergence of the vector at
22. Determine the scalar product clf vectors a and (3,2,1 )
C. A. 9 a. 7 C. 12 D. 15
A. 12 8. 24 C. 36 D. 48
Given a vector V = (x2y)i - (xy)j + (xyz)k.
23. Determine the magnitude of the vector cross 36. Determine the gradient Of the vector at (3,2,1 )
product A x a. A. 12.6 8. 13.7 C. 14.6 D. 15.5
A. 11.2. 8. 12.4 C. 13.6 D. 14.8
Given a vector V = (x2y)i -(xy)j + (xyz)k.
24. At what angle does vector C makes with the x 37. Determine the curl of the vector.
- axis? A. (xy)i ~(yz}j + (x2-y)k
A. 70013' C. 74al2. 8. (xz)i i-(yz:)i + (y-x2)k
a. 72°23' D. 7609'
S: %): : (¥z)i: (%2-+X;)i
25. Find the cross product of vector a x C.
A.15i-3j+18k
8. -15i-3j-18k
C.18i-3j-15k
D. -18i+3j-15k

MPHT Review Center: REF Reviewer i`n MATHEMATICS Page 15


10. How many 4 digit numbers can be formed
by the use of the digits 1 , 2, 3, 4, 6 and 7
if one digit is used only once?
For items 1 ancl 2 A. 360 a.400 C. 320 D. 420
Given the following:
12 34 45 23 87 91 121 11. There are 5 main roads between cities A
and a, and four between a and C. In how
1. Findthemean. many ways can a person drive frc]m A and
A. 54 a. 59 C. 57 D. 61 C and return, going through a on both
trips without driving on the same road
2. Find the population standard deviation. twice?
A. 35.56 C. 37.75 A. 120 8. 240 C. 360 D. 440
8. 33.45 D. 38.21
12. Three boys and three girls sit in a row of 6
3` Find the sample standard deviation. chairs. In how many ways can they sit in
A. 40.78 C. 43.34 alternate seats
8. 41.22 D. 44.34 A. 36 8.18 C. 72 D.144

For items 4 and 5 13. Three boys and 3 girls sit in a row. In how
A Math Professor gives the following scores tc) his many ways can they sit if the three girls
students. are always together?
Frequency 1 2. 3 6 A. 144 8. 152 C. 100 D. 122
Scores 30 42 50 60
14. How many 4 digit numbers can be formed
4. Find the weighted mean of the scores. by the use of digits 1, 2, 3, 4, 6 and 7 if
A. 56.41 C. 56.78 one of the digits is iJs}ed only once in one
8. 52.00 D. 55.34 nl,mber.
A. 360 a. 400 C.320 D.420
5. Compute the standard deviation.
A. 8.12 C. 10.12 15` Four different colored fags can be hung in
8. 9.49 D. 8.33 a row to make a coded signal. How many
signa`s can be made if a signal consists of
6. A student receive his grades as follows: the display of one or more flags?
Subject Grade Units A. 64 a. 68 C. 62 D. 66
Algebra 1.5 5
English 1.75 3 16. How many different permutations can be
Philosophy 1.25 2 made from the letters MISSISSIPPI?
Chemistry 2.5 4 A. 35650 C. 34650
Drawing 1.0 2_ a. 32650 D. 33650
What is the students weighted average?
A. 1.6 8. 1.5 C. 1.7 D. 1.8 17. There are 2 copies each of 4 different
books. In how many ways can they be
7. A club of 40 executives, 33 like to smoke arranged in the shelf?
Marlboro, and 20 likes to smoke Philips A. 2340 C. 2321
Morris. How many like both? a. 2520 D. 3410
A. 12 C. 14
8. 13 D. 15 18. In how many ways can 10 trees be
planted in circular lot?
8. A survey of 500 television viewers A_ 362,880 C. 432,112
produces the following results. a. 3,628,880 D. 442,122
285 watch football games
195 watch boxing games 19. There are 13 teams in a tournament.
115 watch basketball games Each team is to play with each other only
45 watch football and basketball games once. What is the minimum number Of
70 watch football and boxing games days can they all play without any team
50 watch boxing and basketball games Playing more than one game in any day.
50 c]o notwatch anyoftne3games A. 11 8. 12 C. 13 D. 14
How many watch boxing games only?
A. 75 C. 95 20. From the digits 0,1, 2, 3, 4, 5, 6, 7, 8, 9,
a. 85 D. 105 find the number of six digit combinations.
A 210 C. 220
9. A factory building has 8 entrance doors. a. 110 D. 102
In how many ways can a person enter
and leave by different doors? 21. There are 10 points A, 8 ... on the plane.
A. 56 a. 70 C. 64 D. 63 How many triangles are determined by
these points?
A` 120 8` 240 C. 720 D. 144

MPHT Review center.: REF Reviewer i.n MATHEMATICS Page 16


22. A semiconductor will hire 7 men and 5
determined the probability of shooting 5
women. In how many ways can the
out of 8 attempts.
company choose from 9 men and 6
A. 13.21% C. 12.38%
women?
a. 11.44% D. 11.44%
A. 678 a. 216 C. 324 D. 560
34. An items cost distribution has a given
23. In how many ways can a committee
function of the probability. What is the
consisting of 3 mean and 2 women be
expected cost?
chosen from 7 men and 5 women?
Cost jn pesos Probability
A. 350 8. 400 C. 300 D. 200
1 0.2
24. In how many ways can you invite one of 2 0.28
3 0-18
your five friends in a party? 4
A. 32 8. 30 C. 31 D. 29 0.23
5 0.11
A. 2.45 a. 2.77 C. 2.11 D. 2.89
25. The lotto uses numbers 1 to 42. A winning
number uses 6 different numbers in any 35. By investing in a particular stock, a
order. What is your chance Of winning if
person can make in one year P 40,000
you bet one ticket? with a probability of 0`3 or take a loss of P
A. 1/4534568 C. 1/5245786
10,000with a probabilityof 0.7. What is
8. 1/6580668 D. 1/2341668
the persons expected gairl?
A. 5000 c. 7ooo
26. If you roll a pair of dice one time, what is
8. 4000 D. 8ooo
the probability of getting a sum Of 9?
A. 1/9 8. 1/4 C. 1/6 D. 1/3
36. A pack of cards contain 52 cards: 13
spades, 13 clubs, 13 hearts and 13
27. Three boys and 3 girls sit in a row. Fincl
diamonds. Of the 52 cards, 4 are aces
the probability that the 3 girls sit together.
one from each suit. The hearts and the
A. 1/5 a. 2/5 C. 3/5 D. 2/3
diamonds are colored red, the spades
and clubs are black. Four cards are drawn
28. What is the probability of a family with 5
from the pack, each card being returned
children of having 3 boys and 2 girls? to the pack before the next card is drawn.
A. 5/16 8. 1/4 C. 3/16 D. 3/8
Find the probability that all are clubs.
A. 1/4 C. 1/256
29. In a basketball game, the free throw
8. 1/100 D. 1/512
average is 0.65. Find the probability that a
` player misses one shot of the three free 37. Refer fo the previous problem. If 5 cards
throws? are drawn simultaneously, find trle
probability_to get all the 4, aces.
3: 8:£!2 g: g:f!f A. 1/54145 C. 4/54145
8. 2/54145 D. 9/54145
30. A bag contains 3 white balls and 5 red
balls. If two balls are drawn at random in 38. A coin is tossed 5 times. Vvhat is the
succession without returning the first ball
probability of getting 3 heads?
drawn, what is the probability that the A. 0.3125 C. 0.6
balls drawn are both red? 8. 0.125 D. 0.2125
A. 3/14 a. 1/3 C. 5/14 D. 3/7
39. A fair coin is tossed 10 times. Compute
31. One bag contains 4 white balls and 3 the probability of getting at least 7 heads.
black balls and a second bag cc)ntains 3 A. 9/64 C. 7/64
• white
from and
the5 black
secondballs.
bagOne ballisis placed
and drawn a. 11/64 D. 13/64

unseen in the first bag. What is the 40. A coin tossed 10 times. What is the
probability that the ball now drawn from probability of getting 4 tails and 6 heads?
the first bag is white? A. 105/512 C. 13/64
A. 35/64 C. 31/64 8. 51/256 D. 25/128
8. 33/664 b. 19/64

32. Box A contains nine cards numbered 1 to


9, and Box 8 contains 5 cards numbered
1 to 5. A box is chosen at random and a
card is drawn. If the number is even find
1. Forthe complexnumberz= 3-4i
the probability that it came from box A.
a- Find the absolute value.
A. 9/19 C. 11/19
Find the argument.
8. 10/19 D. 21/19 -53. 1 30 C. 7,47.|3o
ngl .130 D. 4,-70.|3o
33. If the probability that a basketball player
sinks the basket at a 3point range is 2/5,

MPHT Review Center: REF Reviewer in MATHEMATICS


Page .17
2. Simplify(2-3i)(5+2i)
1
A. 18-10i C.14-19i A r2i _:1 _:2]
a. 20-15i
cL32 2
D.16-lli -1
3. Simplify: i29+i2i +i
3
A. 3i 8.1-i C.1+i D. 2i 8 I: =o2 _2.I D I-:1 -2
-2
4. Find the principal value ofln(4 +5i)
A.1.86+0.90i C.1.23-0.78i 17. Solve for x and for the given equation
a.1.23-0.56i D.1.67+0.45i
I;I-:I
A. 5,-5 C.15,-15
5. Find the principal value ofjF
a.10,-10 D-20,-20
A. 0.1236 C. 0.1854
a. 0.2078 D. 0.2979
18 Simplify3[:7 ;I t 2 I:
6. (1+i)7is
A. 7+8i C. 3+8i 27 17 -
a. 8-8i D. 4-8i A. 35 5
2845
7. ln(3+4i)is?
A.1.61-0.88i C. 1.44-0.67i
2:J7 2717
a.1.61 +0.93i a. 35 5 D. 35 5
D` 1.45+0.93i
2815 Z845
8. Find the value of x in 3x + 4y + 3yi + 15 -3i =
0 213
19` FindtheinverseofamatrixofA= 61 4
A. -6.33 a.2.33 C.1.33 D.-4,33
372
9. The third principal root of46 -9i
A. 3-4i B.2-3i C.3-5i D.4-3i

10. The first root of (1 + i)1J5 is


A.1.031 +0.13i C. 0.168+1.06i
a. -1.031 +O.23i D.1.231-1.06i

11. Find the value of sin (3 + 2j)


A. 0.531 +3.59j C. 0,731 -3.59j
8. 0.531 -3.59j D. 0.231 -3.19j

12.-Find the`value of cos (1 + 2j)


A. -2.0327-3.0519i
8. 3.0327-4.0519i 20.i:nd3t/::LaB!a8;s|ran8:a:¥/:ff(t)D=t;/s4
C. 2.0327-3.0519i
D. 2.0327-4.0519i
21.:I.nd:;}=e:L=a:);;ceTransformofcos5t

13. Evaluate the determinant:A = 8: :;(::::i,


A. 4 a. 5 C. 6 22. Find the Laplace Transform of t sin t

14. Compute the value of x by determinant: 3: 3,S('s(£2++t;2)2 8: ±;{§2 I :}2


3 -2 4 2
2 1 -3 5 23. Find the Laplace Transform of e3t cos 2t
X= -4 0 -1 -2
-5 3 2 4
A. -385 a. -282 C. -427 D. -126 i. ;i;::3!;|{;s-+3!;2++42)
D. 3s/(s2+4)
15. Evaluate the determinant:
1 14 3 1 24. Find the Inverse Laplace Transform of 3/(s2 -
X=
1 5 -1 3 16)
1 -2 2 -3 A. 3sinh4t C. 4sinh3t
3 -4 -3 -4 8. 3/4sinh4t D. 4/3§inh3t
A. 489 a. 373 C. 326 D. 452
25. Inverse Laplace Transform cif i
16. If A =[-:2 -:1 find the transposed A.1/3-1/3e-3t C. 1/5+ 1/5e3t
8.1/3+1/3e3t D. 3-3e-3t
Matrix AT.

MPHT Review Center= REE Reviewer in MATHEMATI:CS Page 18


26. Find the Inverse Laplace Transform of £=±E
A. 4cos3t+sin3t 11. Evaluate r(8)
8. 4cos3t+ 1/3sin3t A` 362880 c.40320
C. 4sin3t+%cos3t a. 5040 D.181440
D. 4cos3t+3sin3t
12. The expression I(10) is equal to EXCEPT
27. Find f(t) if L(f(t)) =
A. 9! c. tor(9)
(s+2)(s-6) a. 9r(9) D. 9(8)r(8)
A. e.2t+3e6t C. 2e-2±+Sect
8. e-2t+2e6t D. 5e-2t+3e6t 13. One term of the Fourier series in cosine form
is 6 cos20TTt. Write the expression in
28. Determine the Inverse Laplace Transform of exponential form.
(s + 2)/(s2 + 4s + 20) A. 5e020lTt) + 5e{-j2oTr{)
A. e€tcos4t C. e-2tcos2t a. 3e020Trt) + 3e(-j2oTrt)
a. e-2tcos4t D. e-3`sin4t c. 4eo2oTrt) + 4e(-j2oiTt)
D. 6e¢2oTTt) + 6e(-j2oTrt)

14. Evaluate the terms of a Fourier series e¢10Trt) +


e(-jloTTt) at t=1.

1. Find theeigenvaluesof 4 A. 0 a- 1 C. 2 D- 4
(.i i;)
A. 5,2 C. -5,-2 15. Sine function `is said to be
a. 5,-2 D. -5,2 A. Odd function
8. Symmetry function
2. Determine the eigenvalues of the following C. Even function
matrix : D. Unsymmetry function
I;: :I
A. 3,4 a. 2,5 C.1,6 D. 4,1
16. The maximum period for the limits of the
Fourier series
3. Find an eigenvectorof [: A. oo B. 9oo c. |8o° D. 36o°
A- L41] 8. 17. The series expansion x + (1/2) x2 + (1/3)x3 +
(1/4)x4 + ... is equal to the summation
4. How many sets of eigenvectors for the matrix A. I;=ixn C. I:=|¥
123
-2 -1 -2?
314 B. I::=i= D. Z:=i :
A.1 8. 2 C. 3 D. 4
18. What i§ the equivalent function of f(x) =
5. - A inatrix com-posed o`f eigenv'ectbrs.
A. Eigen matrix C. Modal matrix Z: :=o£?
a. Spectral matrix D. Inverse matrix A.ex a. sinx C. cosx D. Inx

6. Find the modal matrixofA 19. Find the Maclaurin series of f(x) = x2ex.
A. I:==oxn+1 C. I;=o¥:=£

B. I:-o# D.I:=o¥
7. A diagonal matrix with eigenvalues. 2o. What is the Goefficjent of x4 term of the Taylor
A. Eigen matrix C. Modal matrix series of cos x at x = 0?
a. Spectral matrix D. Inverse matrix A.1/4 a.1/6 C.1/12 D.1/24

8. Inverse of modal matrix (M) times the given 21. What is the coefficient of (x -1)2 term in the
matrix (A) times the modal matrix (M) is equal Taylor series expansion of f(x) = ln x
to expanded about x = 1 ?
A. Eigen matrix C. Modal matrix A.-1 a.1/2 C.-1/2 D.1/6
a. Spectral matrix D. Inverse matrix
22. Suppose that f'"(x) = i(x) for all values of x
9. The equation (A - ^I)x has zero as a and notice that by differentiating found out
characteristic root if and only if A is that f(4) (x) = f'(x), fi5)(x) = f'(x) ,.... so on and
matrix so forth. Suppose that f(3) = 1, f'(3) = 2, f'(3) =
A. Inverse C. Singular 3` Find the coefficient of the term having (x -
a. Modal D. Diagonal 3)4-
A. 3/2 a.1/6 C.1/12 D. 2/3
10.Ej.ndv#f)t!]Off(t)=t^(-1/2)

8. 112J(TTIS) 3: ?;F';i

MPHT Review center: REE Reviewer in MATHEMATICS Page 19


23. Let f(x) = Vx and a = 16. Find the Taylor
35. He was very jnfluential in the standardization
polynomial degree 2 for f(x) at a. of other mathematical forms and notations. He
A. 4+1/2(x-16)+1/16(x-16)2
named the square root of - 1 with the symbol
a. 4+1/4(x-16)-1/64(x~|6)2
ofi-
C. 4 + 1/8 (x ~ 16) + 1/256 (x -16)2
A. Euler C. Euclid
D. 4+1/8{x-16)-1/512(x-16)2
a. Fermat D. Descartes
24. The series is said to be _ if the limits of
the sum as the number of terms approaches
infinity exists and is equal to the sum Of the
given infinite series.
A, Divergent C. Dissonance
8` Convergent D. Assonance
¥ULF=iELt=ecrHe8t'acnEg!eof|argestareathatcanbe
25. An harmonic series is said to be inscribed in an equilateral triangle of side 20.
A. Divergent C. Dissonance
8. Convergent D. Assonance a. ::!3 g: g8#

26. Find the Fourier series expansion Of 2` Bob is 2 years from being twice as old as
f(x)=0 -5=x<o Ellen. The sum of twice Bob's age and three
f(x)=3 O<x<5
times Ellen's age is 66. How old is EIlen?
Period = 10 A. 15 8. 10 C. 18 D. 20

A. :+ Z:nco=[(=[1 ~ cosnn])sin(¥) 3. What is the maximum area of a rectangle


8. :+I::=L(=[1 -cosnIT])cos(¥) whose base is on the x-axis and whose upper
two vertices lie on the parabola y = 12 -x2.
c. 3+zg=1(=[1 A.16 a. 32 C. 64 D. 8
(-":-:.I)
D. : + 2::=i(±[1 -sinnIT]) cos(¥)
4. Harold used a 3% iodine solution and a 20%
iodine solution to make a 85rounce solution
27. [f n js any positive integer, then n(n -1)(n - that was 19% iodine. How many ounces of the
2)(n -3) ... (3)(2)(1) =
3% iodine solution did he use?
A. e^(n-1) C. n!
A. 5 a. 80 C. 60 D.20
8. (n-1)! D. (n-1)^(n)
5. The manager of a garden store ordered two
28. Given the Fourier series in cosine form, f(t) = different kincls of marigold seeds for her
display. The first type cost her $1 per packet
t5h:°f:£:#)etniazfcr::j::8}).t+COS(80Pi)t.Find and the second type cost $1.26 per packet.
.A. 20 8. 40 C.10 D. 60 How many packets Of the first type did she
" purchase if she bought 50 more of the $1.26
20. The symbol "/ " used in division is called
packets than the $1 packets and spent a total
A. Modulus C. Soljdus Of $402?
a. Minus D. Obelus A. 150 a. 200 C. 250 D. 100
30. The constant "e" is named in honor of: 6. Two angles are complementary. The larger
A. Euler C. Euclid angle is 15° more than twice the smaller. Find
8. Eigen D. Einstein the measure of the smaller angle.
A. 25° 8. 65° C. goo D. 82.5o
31. What do you call a radical expressing an
irrational number? 7. If 3x2 is multiplied by the quantity 2x3y raised
A. Surd C. Complex to the fourth power, what would this
a. Radix D. Index expression simplify to?

32. f is a function such that f(x) < 0. The graph Of 3: i:2X:;#i6y4 8: g##
the new function g defined by g(x) = |f(x)I is a
reflection Of the graph of f 8. Sara's bedroom is in the shape of a rectangle.
A. ontheyaxis C. ontheliney=x The dimensions are 2x and 4x + 5. What is
a. onthexaxis D. ontheliney=-x the area of Sara's bedroom?
A. 18X C, 8x2+5x
33. If f(x) is an odd function, then I f(x) I is a.18x2 D. 8x2+10x
A. an odd function
8. an even function 9. What is the minimum pcissible perimeter for a
C. neither odd nor even rectangle whose area is 1 00 in2?
D. evenandodd A. 10 a. 20 C. 30 D. 40

34. Find the 10th term of the series 1 I 4, 9,16 ,... 10. Patrick has a rectangular patici whose length
A. 80 8. 88 C. 96 D. 100 is 5 in less than the diagonal and a width that
is 7 in less than the diagonal. If the area of his

MPHT Review Center= REF Reviewer in MATHEMATICS Page 20


patio is 195 m2, what is the length of the 20. A 6-foot spruce tree js planted 15 feet from a
diagonal?
lighted streetlight whose lamp is 18 feet above
A.10m 8. 8m C.16m D. 20m
the ground. How many feet long is the
11. Samantha owns a rectangular field that has shadow of that tree?
A. 5 8. 7.5 C. 7.8 D. 9.6
an area of 3,280 square feet. The length Of
the field is 2 more than twice the width. What
21. A 1-inch diameter coin is thrown on a table
is the width of the field?
covered with a grid of lines two inches apart.
A. 40ft C. 41ft
What is the probability the coin lands in a
a. 82ft D. 84ft
sqLlare without touching any of the lines Of the
12. A river is 1 mile wide. Frank wants to get from grid?
A. 3/4 a. 1/2 C. 1/4 D. 0
point A to point a on the opposite side Of the
river, 3 miles downstream. If Frank can run 5
22. In a roomful of 30 people, what is the
miles per hour and can swim 3 miles per hour,
what js the least amount of time in which he probability that at least two people have the
same birthday? Assume birthdays are
can get from A to a?
uniformly distributed and there is no leap year
A- 52min C. 76min complication,
a. 35min D. 27min A. 0.31 C. 0.81
8. 0.71 D. 0.61
13. A garden in the shape of a rectangle i§
surrounded by a walkway Of uniform width.
For problems 23 to 25:
The dimensions of the garden only are 35 by
Let S be the triangle with vertices A = (2, 2, 2), a
24. The area of the garden and the walkway = (4, 2,1) and C = (2, 3,1).
together is 1,530 sqLiare feet. What is the 23. Find the cosine of the angle ABC at vertex A.
width of the walkway in feet?
A. 4ft a.5ft C.34.5ft D.24ft 3: 1;#0 g: i;#1
14. Bill and Ben can clean the garage together in
24. Find the area of the triangle ABC.
6 hours. If it takes Bill 10 hours working alone,
A. 0.7 8. 2.4 C. 1.5 D` 3.2
how long will it take Ben working alone?
A.11 hours C.16hours
25. Find a vector that is perpendicular to the
a. 4hours D.15hours
plane that contains the points A, a, C.
15. A rectangular garden has a width Of 20 feet
3: =?: 1',-6= 3: =:i,26,22'>
and a length Of 24 feet. If each side Of the
garden is increased by the same amount, how 26. Sand is being dumped from a dump truck at
many feet is the new length if the new area is
the rate of 10 ft3/min and forms a pile in the
141 square feet more than`the original?
shape of a cone whose height is always half
A. 23 a. 24 C. 26 D. 27
its radius, How fast is its height rising when
the pile is 5 ft high?
16. Find the rectangle Of largest area that can be
inscribedinanequilateraltriacrgI:oot3Side2°.
3: 8:8?f#:;: 3: 8:83?#:::
A. 45V2
a. 39J5 D. 56wli 27. A standard deck Of 52 playing cards consists
of 4 suits (spaces, hearts, diamonds, and
17. In parallelogram ABCD, m£A = 3x + 10 and
clilbs) of 13 cards each. How many different
m£D = 2x + 30, find the m£A. 5-card hands can be formed?
A. 7oo 8. 4oo C. 86o D. 94°
3: 2:€33:336 8: 3:988:888
18. If the radius of a right cylinder is doubled and
the height is triplecl, its volume is 28. An airplane is built fo be able to fly on one
A. multiplied by 12 engine. If the plane's two engines operate
8. multipliedby2 independently, and each has 1%. chance of
C. multiplied by6 failing in any given four-hour flight, what is the
D. multipliedby3 .
chanie the plane will fail to complete a four-
hour flight to Cebu due to engine failure?
19.The dimensions of a rectangle are A. 0.0199 C. 0.019
Continuously changing. The width increases at a. 0.0001 D. 0.81
the rate of 3 in/sec while the length decreases
at the rate of 2 in/sec. At one instant the 29. A point is moving along the circle x2 +'y2 = 25
rectangle is a 20-inch square. How fast is.its in the first quadrant in such a way that its x
area changing 3 seconds later? coordinate changes at the rate of 2 cm/sec.
A.16 in2/see, increasing How fast is its y coordinate changing as the
8.16 in2/sec, decreasing point passes through (3, 4)?
C. 27 in2/sec, increasing A. 4/3 cm/see C. -3/2 cm/see
D. 27 in2/eec, decreasing 8. -7/4 cm/see D. -2/3 cm/See

MPHT Review Center: REF Reviewer in MATHEMATICS


Page 21
30. A pair of fair, standard dice are rolled. What is
43. For the 2 functions, i(x) and g(x), tables c>f
the probability that the sum of the dice is 5? values are shown below. What is the value of
A. 1/9 8. 2/9 C. 1/36 D. 5/36
g(f(3))?
For problems: 31 to 33
If P(A) = 1/2 and P(B) = 1/2 and P(B/A) = 1/3.
31. Find P(A and a)
A. 2/3 8. 5/6 C. 1/6 D. 1/2

32. Find P(A or 8)


A. -5 a. -3 C. -1 D. 2
A. 2/3 a. 5/6 C. 1/6 D. 1/2
44. Jodi wishes to use 100 feet Of fencing to
33. Find P(AVB)
ericlose a rectangular garden. Determine the
A. 2/3 8. 5/6 C. 1/6 D` 1/3
maximum possible area of her garden.
A. 144ft2 C. 225ft2
34. Find the volume of the solid of revolution
8. 625ft2 D. 96ft2
formed by rotating the region bounded by the
parabola y = x2 and the lines y = 0 and x = 2 45. Find the area of the region bounded by y = x3
about the x-axis` -3x2 + 2x + 1, the x~axis, and the vertical
A. 64TT/15 C. 8Tr lines x = 0 and x = 2.
a. 256Tr/25 D. 32TT/5 A 1 a. 2 C. 3 D. 4
35. What are the odds in favor Of rolling a sum Of 46. Justin earned scores of 85. 92, and 95 on his
seven in one roll of a pair of fair standard science tests. What does he need to earn on
dice? his next science test to have an average
A. 1:6 8. 1:5 C. 6:29 D. 6:36
(arithmetic mean) of 93%?
A. 93 a.100 C. 85 D. 96
36. Find ln(3 + 4i)
A.1.61 +0.93i C.1.01 +3.56i 47. Brad's class collected 320 cans Of food. They
8. 1.23-4.1i D` 1.91-1.23i boxed them in boxes of 40 cans each. How
many boxes did they need?
37. Find the area of the region above the x-axis
A. 280 8.10 C. 8 D. 5
bounded by the function y = 4x - x2 - 3.
A. 1/3 a. 2/3 C. 1 D. 4/3
48. Find the volume of the solid revolution
obtained by revolving the region bounded by y
38. Listed below are 4 functions, each denoted = x - x2 and the x-axis about the x-axis.
g(x) and each involving a real number A. iT/15 C. in/60
constant c > 1. If f(x) = 2X, which of these 4
a. TT/30 D, Tl./10
.... functions y_ields the .greatest value for f(g(x)),
for all x > 1 ?
49. Joey participated in a dance-a-thon. His team
started dancing at 10 A.M. on Friday and
3. g(;)::7x 3: g(x):::ex stopped at 6 P.M. on Saturday. How many
hours did Joey's team dance?
39. If the function f satisfies the equation f(x + y) = A. 52 8. 56 C. 30 D. 32
f(x) + f(x) for every pair of real numbers x and
what are the possible values of f(0)? 50. Callie's grandmother pledged $0.50 for every
Any real number mile Callie walked in her walk-a-than. Callie
Any positive real number walked 9 miles. How much does her
grandmother owe?
0 only A. $4.50
a. $18.00
40. The imaginary number i is defined such that i2
= -1 . What does i + i2 + i3 + . . . + i23 equal?
A. i 8. -i C. -1 D` 1

41. Find the area of the region bounded by y = x2


- 5x + 6, the x-axis, and the vertical lines x =
0 and x = 4.
A. 25/6 C. 16/3
8. 18/5 D. 12/7

42. If the first term in an arithmetic series is 3, the


last term is 136, and the sum is 1,390, wriat
are the first 3 terms?
A. 3,10,17 C. 3,361/3,70
8. 3,23,43 D. 3]69`/2,136

MPHT Review Center: REE Reviewer in MATHEMATICS Page 22


Lq*

So]utions and Answer Keys


• EXERCISE NO. 1 : AIgehra - 1
• EXERcfsE NO. 2: Algebra -2
• EXERCISE NO. 3: Trigonometry
• EXEFtc[SE NO. 4: Plane and Solid Geom'etry
• EXEFtc[§E NO. 5: Analytic Geometry - 1
® EXERcfsE NO. 6: Analytic Geometry -2
• -EXERcfsE-NO. 7: Differs.ntial Calculus
• EXER'CISE NO. 8= integral Cafc,ulu`s
• EXE.RC[SE N'O. S:
• EXEF¥CISE NO,

MPHT Review Center: Solution and Answer key in MATH Page 23


11. By Djscrimjnant: 82 -4AC = 0
(4m)2 -4(4)(4m+5) = 0
mi=-1andff2Lrf

12. By Discriminant:
82 - 4AC = 0
SOLUTIONS TO THE PROBLEM SET k2 -4(3X12) = 0
k--_1_2_
iiiiiiiiiiii'''''''|?['£:I_"tsl:[['_=
13.3ifJfiEFEfFf6fiFF6-=aJJJffJFT=JJJfFJfg6-
= 2((x30)1ql„
1. Byshift-Solve:
=2±
3 + 8x = 35
x_-4
14. (x+y)10, 4th term (r = 4)
n = 10, r -1 = 3, n -r + 1 = 7
2. Byshift-Solve
Formula
6x - 2= 2x + 6
rth term = (ncr.1) xn++1 yr-1
xL=i2 = (10C3) (x4+3)(y3)
=1_20xlyf_
3. Byshift-SolvB
j2o=-x-x 15. (x . y)15, i2th temi (r = 12)
¥4 and x =_i n = 15, r-1 = 11, n-r+ 1 =4

4. Quadratic Equation: A>(2 + Bx+ C = 0


Formula
Use Mode 5 3 and input all coefficients
rth term = (ncr.1) xrm+1 yr-1
and constants = (15C1 1 ) (x4)(. y)11
ab ---sO$5x`vll
11

xi--3a_pdx2---4 16. (x2-12)8, 5th term/middle term (r = 5)


n = 8, r - 1 = 4, n ~ r + 1 = 4
5. Solvethevalueofx
Formula
42x+1 = 1024
rth term = (nc.+1) x"+1 yr-1
x=2 = (8C4) (x2)4(_ 12)4
Then,4X=42=|6 _-4E150_±

6. Iog3(81xp=16
1 7. (x-3y)12, 6th term (r = 6)
x--4 n = 12, r-1 = 5, n -r + 1 = 7
Formula
7,10922+1092X=2
ran rth term = (ncr~i) xur+1 yr-1
Coeff. of rth term = {|2C5) (1 )7{-3)5
---192,456_
8. EnterMode53
abc 18. Sum Of coefficient (SOC)
1 11 28
(a + b - c)8
¥i_---7andx2---_4_ SOC = (1 -1 + 1)8
S a C -- 1
9. A=7; B=2k-1;C=€k+2
-8_C
AA
19. Sum Of coefficients for (ax + by + . . . + k)n
-8=C SOC = (a+b+ .... +k)n -(k)n
- (2k -1 ) = €k + 2 (4x - 3)9 . k= - 3
k-_1 SOC = (4 - 3)9 - ( . 3)9
SOC=€
10. A = 2 and 8 = -k
20. Sum of exponents for (ax + by)n
==4 SOE=¥(exp.ofx+exp.ofy)
¥=4 For (x + y)10,

an SOE = T^ (10)(11 ) (1 + 1 )
Sot = 110

MPHT Review center: Solution and Answer key in MATH Page 24


21. Sum of exponents for (ax + by)n
30. The man -hour js directly proportional to
SOE = ¥ (exp. of x + exp. of y) the production
2 hrs (2 men) 60 men (200 hrs)
For (2x + 1 )7, 1 unit U2
SOE = ./z (7)(6) (1 +0) U2 = ap
SOE = 28

22. Solve for x from the divisor


X-2=0 + x=2
Input the dividend
R = x4 - 2x3 - 3x2 - 4x - 8 1. a2=1x2=2
Press CALC., then enter 2 a3 = 2 x 3 = 6
R = - 28 a4 = 6 x 4 = 24
a5 = 24 x 5 = 120
a6= 120 x 6 = 720
23.x~K=0 +x=K
Then the missing term is ee
R = f(x) = (x + 4)(x - 3)+4
-K = (K + 4)(K -3)+4
2. al=1
EL± a2 = 1 + 3 = 4
a3 = 1 + 3 + 5 = 9
24.x+1=0 +x=-1 a4 = 1 + 3 + .5 + 7 = 16
Solve for A if x = - 1 and i(x) = - 15 a5 = 1 + 3 + 5 + 7 + 9 = 25
A}t3 + 2x2 - 18x + 17 = -15 By inspection, Sn = gL2
A-42
3. Foran=2x-1,
25. Enter Mode 5 4 S5 =Z!=[(2X-1)
ASsuming that the equation is x3 _ 5x2 + S5--31
2x+8
ABCD 4. Sn=2tT+1-5
1 -5 2 8 The sum of the first 5 terms
Xi = -1 ; X2 = 4; X3 = 2 S5 = 25" -5 = 251
CorrectEquatiQn:jr3L±±a1_+2x±L±io The sum Of the first 4 terms
S4 = 24+3 -5 = 123
26. R -k£ a5 = S5 - S4 = 251 - 123
a5 = J2£
-±±'-
RiYi2
XIX2
R2Y22

5. EnterMode32
(loo)(3)2 = gaff
24
R2 = 50

Press AC, then 100 Shift Stat Reg y


27. R = a:
aioo = loop = 9i92
But the area remains constant, then
R1 = kL1
6. EnterMode32
loon = k(10m)
k=10
R2=k(12)=flo

28. By ratio and proportion, Press AC, then 31 Shift Slat Reg a
16 : 12 = 12 : X n = 31£ = 8
161Z
7. EnterMode32
` = TTTTTTTTTTTTTTTTTTTTTTTTTTTTTTTTTTT

12x
an
29, p - ¥
Press AC, then 500 Shift Stat Reg a
(250mw)100km (300km)P2
n = 500£ = 246 terms
(24.0 mw)1,500A 500kv(2,000A)
P_2--230kv
The sum of 246 even integers
z txy, 1 , 246t = Q2rm

MPHT Review Center= Solution and Answer key in MATH


Page 25
8. EnterMode33
X
y 15. Fctr harmonic progression, enter Mode 3 2
1 3 {o solve the arithmetic progression
2 3+5 ai = 2, a3 = 6
3+5+7
3 X y
Press AC, then 2600 Shift Stat Reg fi 1 2
n=2600fti=ioterms 3 6

9. a4=ai+3d a3=ai+2C! Press AC, then 1 1 Shift Stat Reg y


a2 = al + a ail = llS = 22
ln harmonic progression, a" = 7/22
(ai)(ai + 3d) = 70
ai2 + 3aid = 70
16. For harmonic progression,
ai = 1/2, a2 = 1/6
(ai + d)(ai + 2d) = 88 Enter Mode 3 2
(ai2 + 3aid) + 2d2 = 88
70 + 2d2 = 88
d=3
ai2 + 3aid = 70
ai2 + 3ai(3) = 70 Press AC, then 8 Shift Stat Reg S
al=± a8 = 8S =. .3o
ln harmonic progression,
10. Enter Mode 3 6 a8 = 1/30 = Q&
X y
3 20 1 7. Let x = age Of F'eter and y = age of Paul
6 160
Press AC, then 1 Shift Slat Reg y
For their present ages
X + y = 21
al = 1y = §
y=21-x i> Equation 1
3 years from now,
11. ai = 0.05(2) = 0.1
x+3=2(y+3) + Equation2
a2 = 0.1 (2) = 0.2
Combining equations 1 and 2
Enter Mode 3 6
x + 3 = 2(21 -x + 3)
x-15

18. Let a = Beth's age and A = Ana's age


Press AC, then 12 Shift Stat Reg g A + 5 = Ana's age after 5 years
ai2 = 12S = 294an 8 + 5 = Beth's age after 5 years

12. For geometric progression, a„ a2, a3 ,... Eq. 1 : A = a + 5


r=±=± Eq. 2: (A + 5)(8 + 5) = 1.5 (A)(B)
ala2 Combining Eq. 1 and 2
2x+7-=
__
lox-7,
(a + 5 + 5)(a + 5) = 1.5 (8 + 5)(a)
x
Solve for x
23C+7 B-io
x--7_Orx---in 19. Let x = number of years ago and M = 36
36 - x = A
1 3. Enter Mode 3 6
x = 36 - A

36 - x = 2 (A - x)
36 - (36 - A) = 2 (A - (36 - A)
Press AC A=24
The sum of the first 10 terms
i (xp, 1 , 1 0) = 22& 20. Let x = amount of 7% solution
y = amount of 12% solution
14. For infinite geometric series
al For amo`unt of solution,
cO _ T= xty=5 Eq-1
Consider the concentration of solution
7x+12y= 10(5) Eq. 2
1-r
r = 3/4

MPHT.Review Center: Solution and Answer key.`In`M.ATH


Page 26
Enter Mode 5 1
Airplane travel against the wind
abc
500 mi
115- R+C =400
7 12 10(5) 1+(=,)hr
(
x = 2± (amount Of7% solution) Enter Mode 5 1
y = 3 L (amount of 120/o solution) ab C
inB 20007
21. Let x = 0/o concentration of the mixture
ffl ffl
400
F` -- 342.85 in h C = 57.14 mph
A a C A+B+C
10L + 15L + 30L 55L
27. Let R = speed of the boat
25% 35% 10% x
C = speed of the current
For upstream
25(10) + 35(15) + 10(30) = 55x
100 200
x = 19.55 % R-C-
X = 29&

22. Let T = total time it will take to fill the tank For downstream
with all three pipes open loo mi
Sum of individrial rates = Combined rate
R+C- -80
1+(=))hr
(+) rate for the pipe to fill the tank (
( - ) rate for the pipe to empty the tank Enter Mode 5 1
11 abc
--::-:I
a+g+ ( 1 -1 200/3
T = 2.18 hrs 118o
F` -- T3.33 in h
23. For a job to finish, C = 6.67 mph
Lorna's rate = 1/5
Fe's rate = 1/4 28. For motion around a circular path in the
Work done by Loma = (1/5)(2) same direction,
Work done by Loma & Fe = (1/5 + 1/4)(X) D = (Ri - R2)(T)
(1/5)(2) + (1/5+1/4)(X) = 1 For D = 1, Ri = 1/8 and R2 = 1/12
x=4enh_Ogre
(:-i)(T,=1
24. Rate of boy = 1/a T--24min_uap§
Rate Of the man = 1/M
1 29. For © = 0° and H = 8
M
3(:) 60(H) ± 2(0)
111 M-
11
E+ ri = a 60(8) + 2(0)
M-
3(i)+ i-11
- L- 11
86 M = 480/11 minutes
a = 24 hours, M = 8 Aours M_-43Jrfl-
25. Let X = number of days to finish the job 3o. For e = goo and H = 0
Total Man -days = 12 men (13 days) 60(H) ± 2(0)
For 4 days, M-
11
the man days is 5 (4 days) 60(0) + 2(90)
For the remaining days, M-
11
the man days is 8 (X - 4) M = 16.36 minutes = 16'21"
12(13) = 5(4) + 8(X -4)
x~-2frs!ap
T`lme..--
• 26. Let R = speed of the airplane
C = speed of the wind
Airplane travel with the wind
500 2000
R-C- =-..
7.

MPHT Review Center: Solution and Answer


key in MATH
Page 27
9. tan(75)=x/1
1. WhenB=270°,|etA=5o x_-=3.732*_in_
Type the given function: sin (270 -A)
Press Calc. (A?) and enter 5 io. By sine law, a = cb
a= = in
sin (270 -A) = -0.9962 if A = 5
Among the choices, substitute A = 5
For choice A, solve - cos A
-cos(5) = -0.9962
-!E-
A = 140, a = 20, C = 20
y23
sin 140 sin20
y = 43.23 in
Then sin (270 -A) = =£osfA!

2. sinA=4/5andsinB=7/25
A = sin-1 (4/5) = 53.13
8 = sin-1(7/25) = 16.26
Since A is in the quadrant 11
A = 180 -53.13 = 126.87
sin(A + a) = sin(A + a)
sin(A + 8) = sin(126.87 + 16.26) = 3L5

3. tanA=1/3
By right triangle approach
A = tan-1(1/3) = 18.43
h
tan a = 1/2
a525 = sin 2o
8 = tan-i(1/2) = 26.57
h_-=i4.78_in
tan (A -a) = tan(18.43 -26.57)
= -0.142
1 1 . c2 = a2 + b2 - 2abcosc
tan(A-a)=iL„ c2=252+162-2(25)(16)cos(94.1)
c=31
4. cos9=-12/13
© = cos-1(-12/13) = 157.38
Another Solution: by Mode 2
sin 6 = sin (157.38)= 5/13
C = lA£C - 81
But sin © in Qlll is ( -)
c = |23£ 94.1 -16|
Thensine=fl3 c = 30.63 = 37

5. Enterthe equation
12. By Sine Law
2(sin(x))2 + 5 cos (x) +1 = 0
x -- 1 2 a
Solve again the equation and enter 360
x a 240

6. tan(4A) =cot(6A)
1
tan(4X) -
I.:ul''iir
X--9
Enter mode 5 1
7-sin(4.8) -¥ a b C
2£_-=_358.52m cos i 1 8 COS 26 32
;``.
sin 1 18 -sin 26 0
x = 23.87, y = 48.07
i h = x sin(62)
_,
= 23.87sin(62)
h = 21.076
4.8c i With the observers height,
H = h + 1.8 = 22ff
8. Letthe radius beone of the legs of the
right triangle formed and the distance
from the center of the Earth to the top of

:::omo::T!a:n62::i::!potenuse.
a -- 1 9 6 k in

MPHT Review Center: Solution and Answer key in MATH


Page 28
13. Let H be the height of the taller tower A + a + C = 3200 mils
2x + x + 15 + 2x + 15 = 3200
x=30
2x = 60
2x + 5 = 75
The smallest angle
x+15-_45O(%\~-QgQrs

21. A = goo ,a = 8oo, c I goo


By sin - co - op rule of Napier's circle
y = 60 fcz7t40 sin(90 - c) = cos A cos 8
y - 50.35 sin(90 - 80) = cos(50) cos a
h = y + 40 = 90.35 in
B=T4.32--7402O'iap
14. Let x be the slant height Of the 22. For 24 hours, the Earth rotates 360°
Alongimde
360
E=E
Atlme

-=L-
135
360
A time
24
A time = 9hrs
From 7:00 AM (GMT), at the East
7:00 + 9 = 16:00 hours
T.,me.. 4 .-00 P.M.

Enter Mode 5 1 23. From 9 A.M. (GMT),at the West


a b C Alongitude Atime
=_
cos 100 cos 43 9 360 24
sin 100 -sin 43 0 45 A time
jr=70.2m y= 14.7m 5dr = - 24
A time = 3hrs
15. 4 cos y sin y (1.-2 (sin y)2) 9:00 -3= 6:00 hours
= 2 sin 2y ((cos y)2 + (sin y)2_ 2 (sin y)2)
Time: 6..00 A.M.
= 2 sin 2y ((cos y)2 _ (sin y)2 )
= 2 sin 2y cos 2y
24. For oblique spherical triangle,
= sin 4v_ cos (A) = - cos (8) cos (C) + sin (a) sin
(C) cos (a)
16. e = cos -1 (+3 / 2) = 300
1 - tan2e = 1 - (tan (300)) = g±g cos (54) = -cos (54) cos (C) + sin (54) sin
(C) cos (82)

C~-158°18'43"

25. E = A + a + C - 1 80
E = 50t89+120 -1 8o= 78°
Area=- 180
7r (6400Z)(79)
Area =
180
Area = 56,476. 06Z km2
tan A = 50/x = x / 1 10
x -- 7 4 . 1 6 in 35. 50 gradients (360/4cO)
50 grad.ients = 45__degrees
1 8. 90 -e = 2/5 (1 80 -e)
ae. 174 degrees (6400/360)
8 = 300
174 degrees = 3_a_94_mi!S_

19. 360 -x = 5(180-x)


40. For any tangent ourve y = a tan mx. the
x _- 1 3 5
period is "/in, then for y = tan 3x, the
period is zE@.
2o. 36o°= 6400 mils
For triangle, the sum of the angles i§
S = 180 = 3200 mils

MPHT Review center: Solution and Answer key in.MATH Page 29


. - i - .` .` . i-.-. =i.|
13` For the area of the trapezoid
bl + b2
5. For angles of tile polygon, A-h hm
2
Sint = 180° (n -2), Sext = 360° A = 8(54)
Sint = Sext A=4se2_cud
180 (n -2) = 360
r\=4ijfdpe
6. 2Siut=3Sext 14. For the rhombus,
2 (180{n -2)) = 3 (360) P=4s
n=sed9f¥ 56.33 = 4s
s = 14.083
7. Fordodecagon, n= 12sides
d-:(n-3,
d-¥(12-3,
d=54

8. Fordodecagon, n =20sides
d-:(n-3,
d-¥(2o-3,
cl = ap
9. Forapolygon inscribed in a circle
<`
R = 10, n = 6 sides
A-¥sin +,=Lo8J
A-¥sin
A = 259.87
-i:-?--:-i-;

13
9_i_`_
By the similar right triangles,
10. For any polygon with "n" sides 13
n = 8, x = 5 cm
cos 0 -
14.083
© = 22.620
A-
4tan(¥) r = 13 sin © = 13 sin (22.62)
8(5)2 r = 5 in.
A- A = "r2 I iT(5)2
4tan(=) A--7Ifl54ij±
A--err-ed
15. The diagonals of the rhcimbus are
11. Apothem - line segment from the center
perpendicular, then trie angle opposite the
of regular polygon to the midpoint of one Sides is goo
of its sides. The radius of the inscribed
circle di = 12cm
For a Circle insc;ribea in a polygorl, a2 I ZX
n=11sides,R=8 -

A = nR2 tan ;-'--`::L`

A=11(8)2tan(¥)
A--2£ouell
12. For a pentagram (star) inscribed in a
circle,
92y+Pxy!h=ai3Eeantheorem,
r= 6 cm x = 8 cm
d2 = 16 cm

£=;{d;&22=%(12cm)(16cm)

A = 1.123r2
A = 1.123(6)2
A--40.4Z8cm2

MPHT Review Center: Solution and Answer key in MATH Page 30


16. The area of rhombus is equal to the area
of 2 equilateral triangles

y--
£¥+P#2h=94%r2eanTheorem,

The area of the triangle


A = % XY
3o = ./2 (X)(J-)
X=52Jap
y = /TF2 -i = 1 2 cm
20. Let X = distance from 8 to C
PA = 200 in, PB = 80 in, PC = X + 80
PA(PA) = PB(PC)
200(200) = 80 (X + 80)
A=23|`fi:2#60)=(6)2sin(6o) X = 420 in
PC=80+X=-
17. Let a = 5, b = 8. c = 13 and d = X A
Solve right triangle whose legs are di and
8 and the hypotenuse is 13
132 = (di)2 + (8)2 -:-:-:, _-:___ _'--i_-i
di = 10.247 in
Solve right triangle whose legs are d2 and
5 and the hypotenuse is 13
132 = (d2)2 + (5)2
d2= 12 in a

21. The arc length


S = r6 = 3©

P = S + 2r
By Ptolemy's theorem, 9 = 3e + 2(3)
did2 = ac + bd © = 1 rad.
(10.247)(12) = (5)(8) + (13)(X) A = 1/2 r2 e = % (3)2(1 rad)
X -- §..§82 cap A= 4.5
18. For a qiiadrilateral circumscribing a circle 22. The right prism is inscribed in a cylinder
where r = radius Of the inscribecl circle
Aquad = VH
Aquad = Jt-3) (3) (4)(-4i = 12 cm2
s = 14 (a + b + c + d) = 1J€ (3 + 3 + 4 + 4)
s = 7 cm
> Aquad = rs
12 on2 = r ( 7 cm )
r~-1_.714er_rl_
Find the diameter (d) Of the cylinder
d2 = 12 + 12 = 2
19. Property of the Circle: The largest triangle
d = 1.414, r = d/2 = 0.707
inside the semicircle is a right triangle
V = TTr2h
The diameter Of the circle
6.283 = IT(0.707)2h
d = 2(6.5) = 13 cm
h=4m
The hypotenuse Of the inscribed triangle
is 13 cm
23. For truncated prism,
V = AB hAVE
For the base of equilateral triangle, where
x = side of the base
AB = 1/z x2sin 6o
riAVE = 16 (3 + 4 + 5) = 4 cm

1,800 = (% x2sin 60)(4)


x = a n4 c in

MPHT Review Center: Solution and Answer key in MATH


Page 31
24. For a cone inscribed in a hemisphere 30. Regu'ar icosahedron js composed of 20
Vreq = Vhemisphere - Vcone equilateral triangles
S = 20AA
S = 20(1/2) (x2)(sin 60)
S = 20 (1/2)(5)2(sin 60)
S = 216.51

For the cone, h = r


1. ( ~2,6)liesonquadrant|y
Vreq = 2/3 lTr3 - 1 /3 TTr2h
Vreq = 2/3 ITr3 - 1 /3 TTr3
V req = 1/3 TTf 2. To find the distance between 2 points,
Enter Mode 2
25. For wedge cut from a cylinder d=IA-BI
V = 2/3 r3 tan e Let A = 3 + 7i and a = - 4 - 7i
V = 2/3 (3)3 tan 45 d = I 3 + 7i - ( - 4 -7i) I
V -- 1 8 in3
d -- 1 5 . 6 5
26. For similar figures, surface area (S) is
directly proportional to the square Of the 3. d = +t=T--x])2 + (y2 -yi)2
edge (x) 15 = viF2)2 + (3 - (--5-5-}Z
X2 = (1 -0.12)Xi = 0.88 Xi
x=fl
SI
Sz 4. Solve the intersection of two lines
S1 Enter Mode 5 1
=
S2
S2 = 0.7744Si = (1 -0.2256)Si
The surface area is decreased by 22.56% -5 -15
X=30/11 Y=51/11
27. The area exposed abcive the liquid
Zone of the sphere (Z) = 2TTrh
Enter Mode 2
d=lA-BI
LetA=#+=iandB=O
d=,¥+=i,
a = 5.38

5. By distance formula
dl = d2

/Gi`+ 2)2 +ICE L``'95iz=/till 4)a + (2 + 7)i


For r = 7.5 cm, x ~- 3 .66i7 = 1 1 13
Z = 2rty7.5 on)(3 cm)
Z--46_FT__a_cl.cm.
6. Since tlie line segment extend 3 times its
28. For the area of a lune, length
Aha = 4TTr2 (era) k = k = Pip / PIP2 = 4PIP2 / PIP2 = 4
Alune = 4TT(30 Cmys(85/360) Enter Mode 2
AIur`e'-~2,670.35cm2 P1 = -3 + 4i, P2 = 1 -2i
P = P1 + k (P2 -P1)
29. For the area Of a spherical triangle P = ( -3 + 4i ) + 4((1 -2i) -( -3 + 4i))
ITR2E
AST- P = 13 -20i
180 P (13. . 20)
For the excess (E)
E = A + 8 + C -180 7. k=Pip/PIP2=3/5
E = 1 15 + 70 + 92 - 1 80 = 97 Enter Mode 2
T[(12)Z(97) Pi = 2 -5i, P2 = -3 + 5i
AST-
180 P = P1 + k (P2 -P1)
ASIT--243.18cmz P = ( 2 -5i ) + (3/5)((-3 + 5i) - ( 2 -5i))
P=-1 +i
P ( -1.11

MPHT Review Center: Solution and An;tier key in MATH Page 32


The equation wil` be
8. Bymidpointformula,
1/4 (x) + ( -1/4)(y) = 1
X1 + X2
2
-_=±
x + (-2)
16. From the line 4x -2y + 2 = 0
x_-6 -2y = -4x - 2,
y = 2x + 1; mi = 2
9. Forcen{roid, The §Iope of the line passing through ( -3,
Xi+X2+X3 0+2+1 5) is equal to 2

-=--
X= __=_
33 Enter Mode 5 1
fi - il ab
yl+y2+y3 -4+6+4 -35
y- -3+1 5+2
y-2 X=-2/11 Y=1/11
Centroid (ji, P) = ( L2 ) The equation will be
(-2/11 )(x) + ( 1/11 )(y) = 1
10. Enter Mode 6 - 2x + y = 1 1
Mat A, 3 x 3 (in x n) 2- v + 1 1a a
-11
MatA - 31 17. Enter Mode 3 2
E3 -81
Use Shift 5 to enter 0.5 det (Mat A)
1Al = ee

Midpoint of the line segment


1 1 . Use matrix to solve the area
Shift Stat Var i = 5
A=±[ii*f43jngRE] Shift Stat Var P = 1.5
Shift Stat Reg a = 1.5 = 3/2 + mi
A=£[(4-24+4)-(-3+32+5)I
For perpendicular bisector that passes
A = I - 2 5 I = i5 through ( 5, 1.5), its slope is m2 = -2/3
Enter Mode 5 1
12. Use matrix to solve the area
ab
A=:[ifngng¥3RE2ngRE] 51.5
5+1 1.5+(-2/3)
A=±[(3-9-4-4)-(8+9+6+1+2)I X= 412S) Y --612:9
The equation will be
A = I - 20 I = 29
(4/29)x + (6/29)y = 1
4x + 6y = 29
13. By slope - intercept form, y = mx + b
42L+ 6v - zgap
y = 2x - 3
The general form will be Z± 18. For the equation 4x + 2y -2 = 0
Multiply 2 by both sides of the ec|uation
14. Two equations of line are parallel if their
2 (4x + 2y - 2) = 0
slopes are equal
9X:+ 4V - 4L±
y = 3x + 2; mi = 3
6x + 2y = 5
19. Distance of ( 4, 2) to 4x -3y + 5 = 0
2y = - 6x + 5
A(Xi) + Bfyi) + C
y = - 3x + 5; m2 = - 3
Then mi is not equal tc] m2, they are 4if VIE-+Ff2Z2
(4)(4) + (-3)(2) + 5
E2affall_e±
JFFFTIf
15. From the line x -y -2 = 0 d-3
y = x-2, ml = 1
The slope of the line passing through ( 4, 20. A = 5, a = 12, C = k, Xi = 2, yi = 1
0) is equal to 1 A(Xi) + B(yi) + C
Enter Mode 5 1 JF-+rfe2
a (5)(2) + (12)(1) + k
4
4+1 k-4
tr+rf
X=1/4 Y=-1/4

MPHT Review Center: SolutidnandAnswerkeyinMATH


Page 33
21. Distance between two parallel lines
Square bc)th sides
A = 3, a = 1, Ci = -12, C2 = -4
9r2 = 9 -12x + 4x2
C2 -Ci -4-(-12)
d- =_ 9(x2 + y2) = 9 -|2x + 4x2
V_A2 + 8 2 V32 + 12 as_+ 9v2 + 1 2x - 9xp
d = 2.53

22. For the line 2x + y = 12


y = -2x + 12; mi = -2 ...--. =i . ``
The angle bet\^reen the lines
m2 - ml 1. Fora conic 4x2 + 3y2 -8x + 16y +19 =0,
tan e - A=4, C=3
1 + m2ml
m2 -(-2) For A ± C with same sign, the conic is an
t:an 45 - E!!i_pse
1 + (-2)mz
m2 = -0.333...` = -1/3 2. A=1,B=4,C=4
Enter Mode 5 1 - Find the discriminant
ab D = 82 - 4AC
-35 = 42 -(4)(1 )(4) = 0
-3+1 5-1/3 1 The cori.ic -is a E_arabo_la
X=1/12 Y=1/4
3. Foraconic4x2+8x-y2+.4y~|5=0,
The equation will be
A = 4, C = - 1
(1 /12)x + (1 /4)y = 1
x + 3y = 12 For A and C with opposite signs, the
x__+3v-12-=J2 con.iclsanHrmer4_ogle

23. Using the distance of a point to a line,


d2 - dl
4x + 3y - 24
V_42 +Sir
-=-_
JgrTTm
5x-12y+30
4. TheconicisacirclesinceA=C= 1
Convert the equation into standard form
(x ~ h)Z + (y _ k)2 = r2
(x2 _ 8x +1 6) + (y2+1 6y+64) = not +1 6+64
1c3(4x + 3y -24) = 5 (5x - 12y + 30) (x -4)2 + (y + 8)2= -1
The radius is imaginary, the conic is an
52x + 39y -312 = 25x -60y + 150 EmmDtvse±
27x + 99y = 462 5.(Xx22+_y22x-+2TX)I(S¥=3}=£)=31+1+4
9X+33v--154
(x -1)2 +ty -2)2 = 36
24. For y = 3x + 2, mi = 3 For standard form of equation of a circle
For y = 4x + 9, m2 = 4
m2 - ml 8X(i?)22},+R°='=73k32==6r2.
tan 0 -
1 + m2ml
4-3 6. x2+y2+12x-8y+16=O
tan e = A = 1, C = 1, D = 12, E = -8
1 + (4)(3)
h = -D/2A = -12/2(1)
0 - 4.3990 h=6
k = - E/2C = - ( - 8)/2(1 )
25. Enter Mode 2 k=4
d=lA-BI Then the center is at C (- 6, 4}
Where A = 5 i 30 and a = - (8 i - 50)
7. Fx°|S±;2nia;S i°£T2o± :g,uation of a circle
a = 10.14

(x - 3)2 + fy + 5)2 = 42
26. Enter Pal ( - 6, - 8) x2 + y2 - 6x + 1 0y + 9 + 25 - 1 6 = 0
r = 10, © = -126.870 -6x+ I 18 = 0
r = 10, e = -126.87® t 36oa
±__10.8--233.23®_ 8. x2+y2+4x-8y-5=0
(x2 + 4x + 4)+(yz -8y+16) = 5+ 4+ 16
27. Using x = r cose and x2 + y2 = r2 (x + 2)Z +(y - 8)2 = 25
For standard form of equation of a circle
3
CcXFH2}Z8;g__~+k23===5rz.

A = ITR2= TT(5)2
A--78.5
3r + 2x = 3
3r - 3 -2x

MPHT Review Center: Solution and Answer key in MATH Page 34


9. ByTrial anderror,
Find the coefficients
Substitute the given points in the
equations from the choices. Shift Stat Reg A = 0.3 = 3/10
Shift Stat Reg 8 = 0.4 = 4/10
Let x = - 3, y = - 1
Shift Stat Reg C = 0.5 = 5/10
For letter A,
x2 + y2 - 2x + 4y - 1 1 = 0
The equation formed will be
(. 3)2 + (_ 1 )2 - 2(- 3) + 4(~ 1 ) - 1 1 = 0
1±0 y = A + Bx + Cx2
For letter a y = (3/10) + (4/10)x + (5/10)x2
1 0y = 3 + 4x + 5x2
§x2 + 4x - + 3 -- 0
f.23+)2y:i_8tx)2-+18(y_5)`fi=o?_1)+i6=o
12±0
15. EnterMode33
For letter C
x2 + y2 t 6x - 1 8y - 1 0 = 0 X Y
(. 3)2 + (_ 1 )2 + 6(- 3) - 18(- 1 ) - 10 = 0 5 0
0=0
-1 0
For letter D
x2 + y2 _ 8x - 12y + 3 = 0
1 8
(_ 3)2 + (. 1 )2 -8(-3) -12(-1) + 3 = 0
49±0 he value of y if x = 4
Then the equation of the circle passing Enter 4 Shift Sfat Reg P
txh2r°+ugh+{-6x3'_~]`d`S_io-_o y = 4y = £
16. By Square law .for the equation of
10. The center is between the endpoints of parabola,
diameter.
h = 1/2 ( 1 0 + 6 ) = 8
k = % (2 - 4) = - 1
-=-
Xi2

yly2
Xz2

The diameter will be Let x2 = 1/2 (64) = 32 at y2 = 8


D = I A - 8 I by Mode 2 If yl -16,
Where A = 10 + 2i and a = 6 - 4i x2 322
I- ±±!:i -
D = 7.21 188
Then, r = 3.61. r2 = 13 X = 48 in.
By the standard form of' equation of circle 2x = 96 in.
(x -h)2 + fy -k)2 = r2, Then the width of the arch at the bottom is
(x -8)2 + (y + 1)2 = 13 9an
x2 + y2 -16x + 2y + 64 i-1 -13 = o
-16x + 17. 4x2+y2 -16x -6y -43 = 0
(4x2 - 16x) + fy2 ~ 6y) = 43
+ 8x -6y + 25 = 0 4(x2rdx+4)+(yz-6y+9) = 43+1 6+9
= -8x -25
±[4(x~2)2+("2=68]±
+ 9 = -8x -25 -9
- -8x - 16 c¥+(fy£-1
= -8(x + 2)
=8 Fcir standard form of equation of ellipse

12. 2x2 -8x -4y + 16 = 0


¥+¥-1
C (h, k) = ffl
Multiply both sides by ire
1 8. x2 + 4y2 - 2x - 8y + 1 = 0
: [2x2 -8x + 16 = 4yE (x2 - 2x) + (4y2 - 8y) = -1
x2 - 4x + 8 = Zly (x2 -2x+1 )+4(y2 -2y+1 ) = 1 +4 - 1
x2 - 4x + 4 = 2y - 8 + 4 (x - 1 )2 + 4(y - 1 )2 = 4
(x -2)2 - 2ty -2)
=1
LR = 4a = 2
a-= 1 / 2 -- a - 5 =a+==
Major axis: 2a = ±
13. x2+x+y+ 1 =0
x2 + x = - y - 1 19. 9#2 + 4y2 -72x -24y -144 = 0
Since the coefficient of y is ( - ) then the A - 9, C = 4
parabola opens c/o«rr"rard. For the major axis,
a2 = 9, a = 3, 2a = 6
14. EnterMode33 For the minor axis
b2 = 4, b = 2, 2b = 4
2a/2b = 6/4
_-1rs

MPHT Review Cent:er: Solution and Answer key in MATH


Page 35 ELi
20. For standard form of equation of ellipse
h = - D/2A = - 8/2(4)

¥+#
whose major axis is horizontal
h=-1
-1 k = -E/2C = -( -18)/2(9)
k=-1
C (h,k) = 1,2 Center (h,k) = i-_ 7, -_Jj
2a = 6, a = 3. a2 = 9
2b = 4, b = 2, b2 = 4 26.44xx22=gxy2_+t98yx2-+it%yyr=if:9=o

¥+¥-1
4(x - 1 )2 + 9(y -2)2 =. 36
4(x2 + 2x+1 ) -9(y2 + 2y+1 ) = 149 + 4 -9

ife06=36
4(x + 1 )2 -9(y + 1 )2 = 144
_-
(x+1)2 fy+1)2
=1
3616
21 . 4r2 + 25y2 -8x -100y -296 = 0 For the transverse axis,
4x2 - 8x + 25y2 - 1 00y = 296 a2 = 36, a = 6
2 a -- 1 2
£(:2_-12)¥++tit?;(_y22T24=yz8£=296+4+100

¥+#-1
For the major and minor axis
27. 9x2 -4y2 - 36x + 8y = 4

:i;2-_346xy+I)(:y42(;28_y)2;+41)=4ut6-4
a2 = 100, a = 1o 9(x -2)2 -4(y - 1 )2 = 36
b2 = 16, b = 4
(x-2)2 (y-1)2
-1
49
c =Vrm
c =V-100 -16-= 9.165 For the transverse and conjugate axis,
c 9.165 a2 = 4, a = 2
e---
a10 b2 = 9, b = 3
e = a &2
22. 9x2 + 25y2 ~ 36x -189 = 0 c =Jazrm
9x2 - 36x + 25y2 = 189
9(x2-4x+4)+25(y2) = 189+36
c--I-=JT33
9(x - 2)2 + 25(y)2 = 225 c rfe3
e=----
a2
¥+¥-1 e = 1.8

28. For the equation of hyperbola


For the major and minor axis
a2 = 25, a a 5
b2 = 9, b = 3 f-¥=1
A = T[ab = "(5)(3) For the transverse and conjugate axis,
b2 = 4, a2 = 9
A--1rm a = 3 (semi -transverseaxis)
23. 9x2 + 25y2 -36x-189 = 0 ±=L2 (semi -conjugate axis)
For the major ancl minor axis
29. The equation of tlie hyperoo!a
a2 = 25, a = 5
b2 = 9, b = 3 (x-2)2 (y+4)2
-1
916
LR = a5
3¥ = Zgr b2 = 16
LR = 3.6 C(h ) = (2, - 4)
For the equation of the asymptotes
24. For the given equation of ellipse
y - k = ±(b/a)(x - 11)
Since upward asymptote, use ( + )
£+£-1 y + 4 = (4/3)(x - 2)
For the major and minor axis 3y + 1 2 = 4x - 8
a2 = 36, a = 6 4±_ 3v - 29iap
b2 = 32, b = 5.66
30. The eccentricity of the hyperbola
c--+RE pi REiiiiii
a--I-62-5.6C;i-~2 e
aa
Since a = b
The distance between foci is JEER.-=--J5if-
2c=4 e-
aa
25. 4x2 - 9y2 + 8x - 18y - 149 = 0 e=JEZ=1.41
A = 4, C = -9, D = 8, E = -18

MPHT Review Center: Solution.andAnswerkeyin`MATH


Page 36
31 . 4x2 -9y2 + 8x -18y -149 = 0
logioe (2)(x2 + 1)(2x)
4x2 + 8x -(9y2 + 18y) = 149 y'-
(x2 + 1)2
4x]081oe
£{X2++t2)¥+_`!(-y:(¥2)2+=2¥Z|)=149+4-9 y'-
(x+1)2 (y+1)2 (x2 + 1)
-1
36_ 16 9. Bypowerrule,
For the transverse and conjugate axis, y= (x2 + 2)i
a2 = 36, a = 6
bz = 16, b = 4 yl=±(x2+2)±-1(2x)
1
y' - (x2 + 2)_Z(x)
LR = 2b2/a = 2(16)/6
LR = 5.33 y'-
(x2 + 2)i
10. From the equation a:a + y2 = 5,
Let: y = rfeT¥
y,-±(Vg=¥,lx-1
1. Setthe calcula{orin Radian mode For the normal line, in =
1 - cos(x) Using point slope form,
HtiRE
X+O (y - yo) = in(x - xo)
Type £=i§£QS2 then CALC (X?) 0.0001 y - 2 = 2(x - 1 )
=1/2 y-2--2:x-2
2r-_ y i a
2. Set the calcula{orin Radian mode
tan(2X) - 2 sin(X) 11. At t = 4 seconds,
v=D'
X3
CALC (X?) 0.0001 = 3 v-i(2ot+(:+1))It-4
3. Asxapproachesl, v -~ fl9.68 T114S±

0(2 - 1) 12. In Radian mode,


0{2 + 3X - 4)
CALC (X?) 1.0001 = g4 y'=±(4cos(X)+sin(2X))lx=2

4. Asx~infinity
y' = -fi
3X4 - 2X2 + 7 13. y = x2ex
5X3 + x - 3 i--rzxi3!X`+yc2e;x
CALC (X?) 9 x 109 For the point/s of inflection, solve for y"
= 5.4 x 109
Y'--2!f¥+4:orf5X+y2(2crx
As x apprctaches co, the limit approaches y" = ex (x2 + 4x + 2)
infiT'itv_ Set y" = 0
-o = ex (x2 + 4x + 2)
5. Asx+2 ex=0
X3 - x2 - 4 x=O
X2-4 x2 + 4x + 2 = 0
CALC (X?) 2.0001 = 2 By Mode 5 3 where a = 1 , b = 4, c = 2
X1--r_2+Jfz
6. Find the derivative by quotient rule i a -- - 2 - I §2
x2
x+1 12x + JC3
_ (X + 1)(2x)-x2(1) 12 + 3x2
(x + 1)2 12 + 3#2
2x2 + 2x-x2 x = 2 and - 2
y'- (x + 1)2
If x = - 2,
xZ + 2x y = 12 -12( -2) + ( -2)3 = 28
y'- Maximum point fr _fff
(x + 1)2 lf x = 2'
y = 12 -12( 2) + (2)3 = -4
Find the derivative by Product rule Minimum point ~
= ex cos(x2)
= ex(-sin(x2))(2x) + cos(x2) ex
-_ ±__(_cos (x2) - 2x sin f*» For the point Of inflection
y" - 6x
Set y" = 0, then x = 0
8. For derivative of logarithmic function lf x = 0,
y,-i,ogau-¥£ y = 12 -12(0) + (0)3 = 12
Inflection point ~

MPHT Review Center: Solution and Answer key in MATH Page 37


15. From the equation x2 = 16y,
x = 10; y = 20 -x = 10
Let:y-:
£=fifF(10)(io)
y, -±(£) lx-4 -:
For the normal line. in = -2 20. Let x, y = dimensions of the printed matter
Using point slope form, xy = 300
(y - yo) = in(x -xo) y = 300/x
y - 1 = - 2(x - 4)
y - 1 = - 2x + 8
2x_+ y = 9
16. The sum of two numbers is 50,
x + y - 50
y - 50 -x
The product of two numbers
P - x(y)
P - x(50 -x)
P - 50x -x2
50 -2x For the area of the poster
Set g = O, then x = 25 A -(x + 10)(y + 20)
The numbers are 25 ancf 25 300
A - (x + 10) +20
( X
17. For maximum area, the triangle is an Set£=0,solveforx
equilateral x = 12.25 cm
a + b + c = 18 300
a + a + a = 18 y = TF = 24,49 cm
a = 6 on = b = c x + 1 0 = 22.25 cm
y+2o--444asan
A = : ab sine
21` Enter Mode 3 3
A=:(6)(6)sin6o
A -- 1 5 . 6 s a . c in

18. Let x = location Of stake from 10 in post

L_--ji±O+jT3m>+2;2-.-5 - 40 ft
r a 8 ft
dL 2x 2(30 -I)
dx Press AC. then
dv = hSdy
Set dL/dx = 0, then solve for x
At h = 4, dv = 2,
x=an dy = dv / h9
Let x and y = dimension s of the fence dy _- 2.I ap
dy = 0.995 ft/min.
X
22. For the total cost,
C = (Cost per hour)(t)
C = (kv3)(t)
y Since the boat run against current,
C - (kv3)
lEE
x + y - 20 V3
y = 20 - x
C (ks,(i-8
A=xy (V -8)(3VZ) -(V3)(1)
A = x(20 - x) £-(ks,
dA = 20 - 2x (v -8)2
(V -8)(3V2) -(V3)(1)
Set dA = 0, then 0-
(V - 8)a
V -- 1 2 k a h
MPHTReviewCenter:SolutionandAnswerkeyinMATH
Page 38
23. Let

E - EA + EB
E-#+±
E =x2J±I +.fi
dB
,a 8IA
4-dA
IRE1. Set into Radian Mode

dE
:I;\:
(4-x)2

X = t.33 in
•A(
-2
x3
•#) /a: (sin(X))4dx
= 32an

24. Enter Mode 3 3


xh 2. Set into Radian Mode
A) lt
Tr
15 -(7.5)2 cos(X) dx
.I.:
0 0 3
~-9±133975
Tt
-15 -(7.5)2
3. Set into Radian Mode
Tr
2
(sin(X))6(cos(X))4dx
/o
_-0.0184

4. ji2j;2y(x2 +y2)dxdy

/12( f+ry2) dy
- 15 ft
d = 7.5 ft /12( ¥+(2„y2) dy
Press AC, then 14
dv = h,dy /12( 3-K5
ii=iRI
At h = 4, dh = 2,
dv = (4P)(2)
CIV -- -!m 5. /o2/o3/oydxdydz
25. Enter Mode 3 3
xh A
0 0
10 I, 10 2
20 0

6. By inspection,

i;§{t§€:;==a3u:i,i::a,3{:du,,ddxx\4x]
Then,
Press AC, then f34*dr-~3an4in8ilife
dv = h9dy 7. d(tan u) =sec2u du
At h = 5, dv = 3, d(cos u) = - sin u du
dy = dv / h9
Fly_-Slay (2sec2x -sinx)dx
Cty=-OQEL2732in/see 2 tan x - ( - cos x ) + C
26. For R = 6 ohms,
2Jtan x + cos x + C
E=IR
1.2 V = (I)(6)
8. /xZVREdx
Set limits 0 to 1
I = 0.2 A
Differentiate E = IR
dE = ldR + Rdl I:x2Jmdr=o.6z3
0 = (0.2)(0.03) + 6(dl) Among the choices, using the same limits
dl = i|±007 A/see

MPHT Review Centel.: Solution. and Answer


key in. MATH
Page 39
For : (x3 + 3):,
13. Solve for the intersection of x = y2 -2 ancl
=:((13+3)2-(o3+3):)=0.623 X _- - y
-y - y2 - 2
Then,
I x2JaT5 dr -_ 2ue9_ {* + 3|3i2 + c
y2 + y - 2 - 0
y - 1, -2
9. I y3f tizTTldy A = J( XR -XL)dy
Set limits 0 to 1
A = /_12( -y -Or2 -2))dx
A = 9/2
/o[y3/ZFTdy=0.3797
Among the choices, using the same limits
For i (2y3 -1)2(3y3 + 1), 14. Solve for trie intersection of y = x3 - 4x
and y = x2 + 2x
= i /(2 - 1)2(3 + 1) - (-1)2(1)\
2(1,) x3 - 4x = x2 + 2x
£((2-1,3 x3 - 4x-x2 - 2x = 0
- Error
x3-x2 -6x = 0
For ± (2y2 -1)±(3y3 + 1), By Mode 5 4 where a = 1, b = - 1, c = -6,
d=0
-±((2-1,:(3+1,-(-1,!
1)Z(3 + 1) - (-1)i(1) X=
b
- Error A- ( Yu -YL)dx
For±(2y+1)±(3y2+1),
A1-f2(X 3 -4x -(x2 + 2x))dx
-i((2+1,i(3+1,-(1
(2 + 1)Z(3 + 1) - (1)¥(1)
= 0.1 976
A.+--tt/3
For ± (2y2 + 1)3(3y2 -1), A2-/n3 ( x2 + 2x ~ (x3 -4x))dx
1 33
A.2-&R,/4
55 ( (2 + 1)a(3 -1) -(1)Z(-1)
-rJ.%J9i7 A=A±+A2=L6/3+ee>|4
Then A--2an
I y3 2ry2+1dy 15. Enter Mode 7
= i i30 (2V2 + 1}3'2(3v2 -|} + C
f(x) = 4 sin(2x)
Start? = 0
10. Let u= 1/x, du =(-1/x2}dx End? = 360
Jsin u du = -cos u + C
p?-15
X(e) f(X) (R)
(: 0 0
-x2du
/ sin(u)
x2
- - (- cos u) i- C
--/sin(u,du 30
45
3.46
4
60 3.46
- cos (1lx' + C 90 0
135 4
1 1 . Let u = y4 and du = 4y3 180 0
feu du = eu + c 225 4
ey4 4y3 dy - eudu = eu + c 270 a
315 4
ey4 +c 360 0

12. Solve for the intersection Of y = 2x, y = x3


Set into Radian mode,
For one leaf, the limits is from 0 to TT/2
2x - x3
x3 - 2x - 0 A=±/r2do=£/o=(4sin(20))2do
x(x2 - 2) - 0
x~_0,rfe2,rf2 A-2LT7.

A = J-( Yu -YL)dx 16. Enter Mode 7


f(x) = 4 (1 + cos(x))
-x3)dx Start? - 0
End? -360
Step? - 15
x3 -'2x)dx

A2 - 1 + 1

MPHT Review center: Solution and Answer key in MATH Page 40


Xe fxR
0 8 ycdA
30 7.46 yu+yL
45 6 dA
2
60 4 9-x2
90 2 dA=/¥(9-xZ)dx
135 0.5
180 0 y= (9 -x2)2 dx
225 1.2
270 H-E±ra
4
315 7.46 20.
360 8
A- (ex) dx = 1.718
The limits is from 0 to IT/2 for one region
r2 = 4(1 + Cos(0)); ri = 4

A = :J(r22 -rt2) d6 1.718iE - xex dx


+ cOs{O)))2 _ 42) de
_x=Qiap ol

2(22.28) 21. Using vertical strip (dy)


y2

17. x2 +y2 = 25 At
2x + 2yy' - 0
x + yy, - 0 (xz -xl)dy
y
Or')2 -
But y2 = 25 - x2,

(y,)2 = 25 -x2
x2
Afryiry--

32
/ ycdA

Ty I:i(4-yg)dy
/_1 + (jj5Z dx P-15
22.. The moment of inertia with respect to y -
axis
S = 1.419 y-+Exfx
ly - J' x2 dA
18. r = 4sine Iy=/o£X2fy)dx

£-4cose y -- ]o. xz (J® dr


©, -0, ©2-: ly -gH
s - Jab r2+(£)2do 23. The moment Of inertia with respect to x -
lt axis
S - /oZ (4 sin(o))2+(4 cos(e))2 do
S = 2IT
Ix= y2dA
TheiFtal length of the curve
ST=2(2TT)=i77' Ix- y2 (x)dy
Ix- yzcJEry§ryDdy
Tx-_a.5Ti4OrL4a
24. The moment of inertia with respect to y -
axis
yl
/ xcdA

18EZ x( 9 -x2)dx x2 (a -vapdx


_x - a a 03
Iy- Qan

MPHT Review Center:. Solution and Answer key in MATH


Page 41
25. By Shell method, 30. The area cif the qijarter circle
V - 2TT

x2
/ x(yu -yL)dx
A-¥-¥-¥n
4
yu -a, yL -0 The centroid of the quarter circle
=-y-£-#-2.12
V - 2Tr r = 10 - 2.12 = 7.88
V - 2TIrA
V = 50.265
V = 2Tt(7.88)
(¥")
26. By Disk method, v=99z2_=rs
V-Tr (y2)dx
/` 31 . For the work done in pumping out water
For hyperbc>la y = 6/x, W=yvE where y = 9.81 x 103 N/in
For hemisphere
V=mf:(=)2dx
v==r3
V--28.Z7
V=¥(3)3=18H
27. By Ring Methc)d,
V-1T
/ (yu2 -yL2)dx h=F-_-i
3R 3(3) 9
88
yu = 2X + 3; y[. = x2 W = (9.81 x 103)(18ft)

For the intersection, w = Ge n4 x 1 a S j


2x + 3 = x2
x2 - 2x - 3 = 0
By Mode 5 3, x = - 1 , 3
V-" 2x + 3)2 -(x2)2) dx
/_31(( 1. The eeeLr of a differential equation is the
V=2_27.97_
hjdrfres+es± occurring deii`rative in the
28. For the functions differential equation
yi=x2:2|8.y2=2
y2-yl-2-:2- (¥)3+3y(g)7+ys($2=5x
Order Of the diferential equation = 2
Solve for their intersection,
x2
2- 2. The ±!egiree Of a differential equation is
ir the Eg!±er Of the ±±giv£§f occurring
x - 4, -4
By Disk method, derivative in the differential equation

/
V-T, (y2)dx
(¥)3+3y(g)7+ys(g)2=5x
Order Of the differential equation = 2
v="/o4(2_¥)2dx Degree Of the differential equation = £
V--2an
3. yor")4+ 2xfy')5 = 8xy2
29. The area Of ellipse
A = 1Tab y(¥)4+2x(35=8xp2
9x2 + 4y2 = 36
Order Of the differential equation = 2

:+f-1
a2 = 9; a = 3
Degree of the differential equation = ±

b2 = 4; b = 2 4. DifFepentiatethe equation: x3 -3xzy = o


A = TT(3)(2) = 6TT 3x2dx -3[x2dy + y(2xdx)I = 0
For center (0,0) 3x2dx - 3xzdy -6:ii;ydy = 0
A(Xi) + Bcyi) + C
Divide both sides by 3x,
ul AI2 + FIE xdx - edy - 2ydx = 0
(3)(0) + (4)(0) -20 (x-2viclx_I_xdv--0
-/__32_ + (4)_2_

r=4 5. Solve for the first and second derivatives


V = ZT[rA = 2TT(4)(6TT) y = cLe-2x + cze3x
V--4J3.14or48TF y' = -2c.e-2X + 3c2e3X
y" = 4cie-2X + 9c2e3x

MPHT Review Center: Solution and Answer key in MATH Page 42


From Eq. 1
Then`
c2e3x = y -cLe-2x Eq.4 |n (1 +y2) = In x + ln 5
Substitute Eq. 4 jn Eq. 2 1 + y2 = 5x
y' = -2c[e-2X + 3[y -c]e-2x]
Yi__ 5x-1_
y' = -2cre-2X + 3y -3c[e-2x
y' = 3y -5cte~2x 10-Let: y =wx
5cte-2X = 3y_y' Eq-5 dy = wdx + xdw
Substitute Eq. 4 in Eq. 3 9x2dx + 3y2dx - 2xydy = 0
y" = 4c]e~2X + 9[y ~ c[e~2X] 9x2cix + 3w2x2ax _ 2x(wx)(wdx+xdw) = o
y" = 4cte-2X + 9y -9c[e-2x 9x2dx + 3w2x2dx - 2\^¢x2dx - 2wx3dw = 0
y" = 9y -5c±e-2x Eq.6 9dx + 3w2dx - 2w2dx-2wxdw = 0
Substitute Eq. 5 in Eq. 6 (9+w2)dx - 2wxdw = 0
y" - 9y - [3y-y']
y' -9y -ty+i By variable separable,
if' -dy +i dx
y==y'-6y_=_a / X /2wdw
9+w2
ln x -ln (9+w2) = ln c
6. Forequation of straightline,y = mx + b e]n6i= = elnc
Since in = b, X
y=mx+in - Eq.1 9+w2
dy = mdx - in = dy/dx
Substitute in in Eq. 1
y = (dy/dx) x + dy/dx
ydx = xdy + dy Multiply both sides by x2
ydx = (x+1 )dy rfec{9x2+_£±
yde2{-{x+iidv-I__9__
11. For differential equation, Mdx + Ndy = 0
7. For the circles with center on thex-axis,
(x - h)2 + y2 = r2
aMaN
--L-
ayax
x2 - 2hx _ h2 + y2 = r2
Differentiate the equation to solve for y' A. (x+y)dx+(x-y)dy=0
2x -2h + 0 + 2yyJ = 0
-1
2x - 2h + 2yy' = 0 %--a¢%y)
6y
x - h + yy' = 0

Solve for y"


%---1 ax
= a is an
1 + [yy" + y'(y')I = 0 EXACT differential equation
1 + yy" + (y')2 = 0
vv'_' + (v'}2 + 14 12. By variable separable.

8. (1 +y2)dx+(1 +x2)dy=O ff=+f%-fo


dx ln (1 + x2) + ln y = In C
rfe+#-o
Integrate both sides
(1 + x2)y = c
Jdix2v_=_£
t¥n-1x+tan-1v_-_±
13. A differential equation is ±g!zgi/ if the
9. 2xydy=(1 +y2)dx derivative of y is independent.
Divide both sides by x, Then, Jar_?'_- 2y coax = sfq¥ is a nonlinear
2ydy = (1 +y2) (dx/x) differential equ.ation

Separate variables and integrate 14. For differential equation y' + Py = Q

/ 1+y2 -/¥
2ydy y' - 3x = 6 where P I - 3 and Q = 6
The integrating factor
|n (1 +y2) = ln x + ln C V = e/Pdx = eJ`-Sax
If x I 2, y = 3 V -- e - 3 x
In(1 + 32) = In 2 + ln C
C=5 15. y' - 3x = 6 where P = - 3 and Q = 6
The integrating factor I = e -3x

MPHT Review Center: Solution and Answer key in MATH


Page 43
The solution
If the roots are real and equal,
y( V ) = + Q ( V )dx + C
y = c[emx + c2xemx + c3x2emx
ye - 3x = I 6e - 3x + C Since in = 1
ye-3x = _ 2e-3x + C
y = ciex + c2xex + c3x2ex
lf x = 0, y = 6,
6=-2+C + c=8
y=e_F_{ci+xc2__+ire
Then,
20. Enter Mode 3 5 where x = time and y =
ye-3x = _ 2e-3X+ 8 amount of strands of bacteria
rL=__- 2 + 8_e_3=

+G . y + lay -- nyA
P = x, Q = x, n = 2
Press AC
Divide both sides by y2
Initially, the number of strands Of bacteria
y:2if +yty-1 =y` Press o then Shift Stat Reg S =
Let z=yl-n=yl-2=y_1
C)9 = en
i: -- -y-2.y
y--i
y-2
21. Enter Mode 3 5
Let x=timeinminutes,y=T-Troom
ln terms of z, the given equation will be T = temperature of the bar
- z' + xz = x
Troom = 0
z' - xz = - x Find x if y = 25 -0 = 25
where P = -x and Q = -x
Solve the transformed linear equation y
1 00 - 0
V = ef -xdx = ex2/a
50-0
zv-/ Qvdx + C Press AC,
Then 25 Shift Stat Reg ft =
r'#Z
-# e _Td# + C
zex±-I-xe-x±dr+c 25S=33gi±_Baplapte
-Jr2 -Jr2
ZGT = eT + C 22. At vo = 0
-x2 h = Vat + 1/2 gt2
Divide both sides by eT 100 = 0(t) + % (32.2)t2
z = Cex^2# + 1 t = 2.5 seconds
But z = y-`'
y± cex^212 + 1_ 23. Let: Q+ amount of salt
dQ
- Qln - Qout
17. y" - 3y' + 4y = 0 E
E -1±(3±)-ri(3=)
In terms Of differential operators, dQ
(D2 - 3D + 4)y = 0
m2 - 3m + 4 = 0 dQ
- 3 - 0.03Q
Solve by Mode 5 3 E
dQ
m=:±¥i
in + 0.03Q - 3
a = 3/2 = 1.5; b = ¥ Solving linear ec|uation; P = 0.03, F` = 3
Solution: y = eax (ci cos bx + C2 Sin bx) v = eJ.pdt = eJo.03dt
V = e!idf rm
y=el.5X[c]cos¥x+Czsin¥X]
Q:v = I Rwdx+ c

18. For s.olutions e-2X and sin 3x,


y = cie~2X + (c2 cos 3x + c8 Sin 3X)
Qeo.03t -
/ 3e°.°3tdx + c

The roots (in) are - 2, 3i and - 3i Qe°.03t = 3eo.03t


(±)+c
(in + 2)(in + 3i)(in - 3i) = 0 Qe°.°3t = |00eo.03t + c
(in + 2)(m2 + 9) = 0 Q = ce-0.03t + loo
m3 + 2m2 + 9m + 18 = 0 Solve C if t = 0, Q = 1
Transform into differentials, 1 - ceo + 100
1L::L±_2V"+9V'+18v--0 c = - 99
Q = -99e-O.03t + loo
1 9. (D3 - 3D2 + 3D - 1 )y = 0 lf Q = 2, solve t
m3 _ 3m2 + 3m - 1 = 0 2 = -99e~0.03t + loo
Solve using Mode 5 4, the value Of in is 1 t--0_.388.:min

MPHTReview.Center:.Solutionand.AnswerkeyinMATH Page 44
7. A=3i+j,B=-2i+4j
24. Given the curves: y = cx2 Find iinit vector A - a
c=ylx2:2 A -8 = (3 -(-2))i + (1 -4)j
Differentiate the equation A - a = 5i - 3j
|A -8| = /EE-+ (=g5Z = V§Z
A-B
lA -BI
5
u -7=i-±j
By orthogonal trajectory,
dy-x 8. A = 3i + 2j, 8 = 2i + kj where k = scalar
= a.) lf A and 8 are orthogonal, A . a = 0
dx2y
2Jf diN = -yJf ]ix` (3i.2i) + (2j.kj) = 0
3(2) + 2k = 0
f2ydy+frdx=fo 6 + 2k = 0
k -_ - 3
yz+f-k b.) lf A and 8 are parallel, A = cB
q.5x2+y2--k (3,2) = c (2,k) = (2c, ck)
3=2c 2=ck
c= 2 = (2/3,k
k_--4!_3_

For # 1, 2, 3, 4 and 5 9. A=-5i+j,B=4i+2j.


Set the calculator in Degree Mc)de (D) a.) By Dot Product
E.nter MODE 8 A.B -5(4)+1(2) -18
_I
Choose Vct A (1 ), in = 2 (2) |A| --V52+12 JT€6
DISPLAY + [1,2] b.)
Shift Vct (5) Data (2) -5(4) + 1(2)
Choose Vet a (2), in = 2 (2) 52 + 1Z
(5i + j)
DI SPLAY + [3,4]
AC 45. 9.
1. Shift, hyp, Shift, Vet (5), Vct A {3), = EI_irJ
AlbscNctA:)--2fl
10. Using Mode 8
2. Shift, hyp, Shift, Vet (5), Vct a (4), = Input the Vct A, 8 and C
Abs (Vet 8) = § VctA[0,0] VctB[3,4]
Vct C I 8, 0]
3. Shift,' Vet (5), Vct A (3), Shift, Vet (5), Dot Solve using Shift 5
(7), snif[, vet (5), vet a (4) For a triangle,
VctA.VctB--1fl AA=±|(VctA-VctB)X(VctA-Vctc)I

4. Shift, Vct (5), VctA (3), x (multiplication AA = 16


sign), Shift, Vct {5), Vct 8 (4)
VctAxvctB = [0, 0, -2] Aparalle|ogram = 2AA
Aparallelogram = 2(16) = £L2
5. By Dotproduct
A . 8 = |A||B|cosO 11. P (-3,4,-1 ),Q (2,5,+)
cose = A . 8 / |A||B| IPQl= (X2 -Xi)2 + ty2 -yi)2 + (Z2 -Zi)2
Vct A.Vct a + (Abs(Vct A) Abs(Vct a))
lpQl= (2 + 3)2 + (5 -4)2 + (-4 + 1)2
cos e = 1 1 + ((2.236)(5)) lpQl= ife
a--10-3050_
12. A = 5i -2j + 6k, a = 8i -fi -4k
A + 8 = (5 + 8)i + (- 2 -5)j + (6 -4)k
6. P=(-1,8),Q=(3,2)
A + a = 13i ~ 7i + 2k
EQ = Q - P
Fa = <3 _ t . 1 >, 2 _ 8> 13. A = 5i -2j + 6kf 8 = 8i -5j -4k
•Fa = «, - 6= A - a = (5 - 8)i + (- 2 + 5)i + (6 + 4)k
A - a -- - 3 i + 3 i + 1 0 k

MPHT Review center: Solution and Answer key in MATH Page 45


22. ByDot Product (Shift, Vct (5), Dot (7))
14. A = -4i + 7j -2k
Vct a . Vct C = 24
3A = 3(- 4j + 7j - 2k) = ±2£_+ 27£=£Lfr
~ 5A = -5(-4.I + 7} -2k) = 290i-35i + 1_OQ±
23. By Shift, hyp and Shift, Vct (5)
|A x Bi = Absevct A x vc.t 8)
15. R (2,-1,3), S (3,4,6)
lA x 81 = rm
|is| = (3 - 2)i + (4 + 1 )i + (6 -3)k
lEl - i + 5j + 3k 24. Solve for the unit vector of C by Shift, hyp

lisl - - = rfe
A-B
and Shift, Vet (5)
uc = Vct C + Abs (Vct C)
uc = [0.272 0.153 -0.136]
IA -Bl
i + 5j + 3k
The angle of Vct C with x - axis
V5E
cos ex = ux
16. A (3, 4, -6), a(4, -7, 8) cos ex = o.272
d =A-B ex--74.22:=zgng
d- (3 -4)2 + (4 + 7)2 + (-6 -8)2
d-&3 25. By Shift, Vct (5)
a x C = Vct a x Vct C
a x C = Shift 5 4 x Shift 5 5
17. C (5, 7, z), D (4,1, 6)
Bxc-_-_1_8i+3i-_]_se
d=C-D
7.28 - (5 -4)2 + (7 -1)2 + (z -6)2 26` Scalar projection of a on A
z-2 B.A VctBDotvcl:A
IAl Abs IVct A)
18. By Dot Product -3m
A . 8 = (Xy)(yz) + (2yz)(2zx) + (32x)(3xy)
A . a = (xy2z) + (4xyz2) + (9x2yz) 27. Vector projection of C on 8
If x = 1 , y = 2, z = 3, C.B_ Vctc.VctB
A . 8 = (1 )(2)2(3)+4(1)(2)(3)2+9(1)2(2)(3) 8-
1812
A . 8 = fl8 = Vcr C Dot Vct a x .Vct 8 + (Abs (Vct a))2
=-±±±3i`+4.163i.+2.2B_7_±~
19. Mode 8
VctA,m=3 [1 46] 28. using Mode 8
Shift Vct (5 Data Input the Vet A, a and C
VctB,m=3 [2 35] VctA[3,1,2] VctB[4,-2,1]
Vct C I 1 , 1 , 3]
AC
Shift 5 3 x Shift 5 4 Solve using Shift 5
VctAxVct8--2.I+7i~5k
AA=±|(VctA-VctB)X(VctA-Vctc)I

For #20 to 27 AA - se
Enter Mode 8
VctA,m=3 [1 3 For # 29 to 32
Shift, Vet (5), Data (2) Mode8,VctA,m=3 [2 4
VctB,m=3 I-1 4 Shift Vcr(5) Data
Shift, Vct (5), Data (2) VctB,m=3 [1 -5 2]
Vctc,m=3 {2 7 AC
AC
29. By Shift 5,
20. By Shift, hyp and Shift, Vct (5) R=A+B
R = Shift 5 3 + Shift 5 4
|R| = Abs(Vct A + Vct 8 + Vct C)
F3=[ 3 -1
lRI - If
I+ -- 3i - i + E_B_
21. By Shift, hyp and Shift, Vct (5)
30. Ft = 3i -j + 5k
|AB| = Abs(VctA-VctB)
Abs = 3 [R| ='Abs (3i -j + 5k)
lRl = ee

MPHT Review Center: Solution.and..Answer key in MATH Page 46


31. Scalar projection of A on 8
By Dot Product, vxv-#l+¥j+¥k
A.B VctADot:VctB
iBi - Abs--tTtry
--2.19
-¥k-¥i-¥j
V x V = xzi + 0 - yk - xzk - 0 - yzj
vxv-->ca2ef-vzi-(v+rf}_±_
32. Vector projection of 8 on A
B.A VctB.VctA
Th=A - Abs(Vet:A)
= Vet a Dot Vct A x Vct A + (Abs (Vet A))2 rs. .-" : . .i-.
---_0.828i-1.665i-1.241_±_
1. EnterMode31
33. Mode 8 Input all the values 12, 34, 45, 23, 87, 91,
VctA,m=3 [24 -8 6] and 121 in the column X
Shift Vct(5) Data AC Shift Stat Var =
_x = 5 9
VctB,m=3 [4 12 6]
AC
2. EnterMode31
By Dot Product Input all the values 12, 34, 45, 23, 87, 91,
A . 8 = |A||B|cos© and 121 in the column X
Vcr A Dot: Vct 8 AC Shift Stat Var ox
cose - c" = €7 I 5
Abs (Vcr A)Abs (Vct a))
cose = 0.0989
3. EnterMode31
e --84.320 = 84pi9i 27':
Input all the values 12, 34, 45, 23, 87, 91,
and 121 in the column X
34. By Dot Product, the angel between A and
8 is.90
AC Shift Stat Var sx
A . 8 = (2i.4i) + (bj . + 2j) + (k . -2k)
sx_-40J8
cos (90) = 2(4) + b(-2) + 1(-2)
4. Entel.Mode31
b=3
To ON the froquency, Shift / Mc]de / Dc)wn
35. For divergence of v Key / Stat / ON
X FREQ
v.v-¥+¥+¥ 301

v.v-¥+#+¥ 422
503
606
Ti.vi-_2ny-x+ry
AC Shift Stat Var X
_x _- a a
At x = 3, y = 2, z = 1,
V.V=2(3)(2)-3+3(2)=|5
5. EnterMode31
36. For gradient of V To ON the frequency, Shift / Mode / Down
Key / Stat / ON
-=gi+#j+¥k X FREQ
301
w=api+2E
ax -' ay j+¥k
422
503
W = 2xyi - xj + xyk
606
At x = 3, y = 2, z = 1,

lvvl--
VV = 12i -3j + 6k

1VV1-&7 6.
AC Shift Stat Var c*
Crx = 9&

EnterMode31
To ON the frequency, Shift / Mode / Down
37. For curl of V, Key / Stat / ON
X FREQ
VxV- 1.55
1-75 3
1.25 2
2.54
12

MPHT Review Center: Solution and``Answer;.key in MATH


Page 47
AC Shift Stat Var X 9. Two different doors use forentry and exit
-x -- 1 - 7 0 3
No of ways to enter by one door = 8
No. of ways tc} leave by another door = 7
7. Let X = like to smoke both Marlboro and N = (8)(7) = ff
Philips Morris
33 -X = like to smoke Marlboro ONLY 10. If 1 digit is used only once using 6 digits
20 - X = like to smoke Philips Morris N=6x5x4x3
ONLY N = 360
By Venn diagram,
|ut PM 11. From city A to city 8,
n = 5 roads
r = 2 different roads used back and forth
From city a to city C,
n = 4 roads
r = 2 different roads used back ancl forth
N = (5P2)(4P2)
N -- 2 4 0
(33 ~ X) + X + (20 - X) = 40
X=13 12. The positions of boys and girls sitting in
alternate seats
8. Letx= no ofviewerswhowatch both BGBGBG
games G a G 8 G 8 2ways
Football BOxing No. of ways to arrange girls = 3!
No. of ways to arrange boys = 3!
N = (3!)(3!)(2)
N=72

13. The positions of boys and girls where the


girls are always together
GGGBBB
BGGGBB
BBGGGB
8 8 8 G G G 4ways
No. of ways to.arrange girls = 3!
45 - X = Viewers who watch football and No. of waysfto arrange boys = 3!
basketball N = (3!)(3!)(4) = ffi
70 -X = Viewers who watch football and
boxing 14. n = 6 digits
50 -X = Viewers who watch boxing and r = 4 different digits
basketball N=6P4=ELO
45 - X + 70 - X + 50 - X = 1 65 - 3X
15. n = 4 different flags
Viewers who watch football only r = 1, 2, 3 c>r 4 flags used for signal
= 285 - (45 - X + 70 - X + X) = 170 + X
Viewers who watch basketball .only N = 4P1 + 4P2 + 4P3 + 4P4
= 1 1 5 - (45 - X + 50 - X + X) = 20 + X N=64
Viewers who watch boxing only
= 195 - (70 ~ X + 50 - X + X) = 75 + X 16. n = 11 letters
170 + X + 20 + X + 75 + X = 265 + 3X l's = 4, S's = 4, P's = 2
n! 11!
Viewers who watch any of the 3 games N=
p!q!r!... (4!)(4!)(2!)
= 500 - 50 = 450
N = 34.650

265 + 3X + 1 65 - 3X + X = 450 17. n = 2(4) = 8


X=20
p = 2, q = 2, r = 2, s'= 2
n!8!
Viewers who watch boxing only
5TFT = (2D(2!)(2!)(2!)
75 + X = 75 + 20 = 95 N~-2L539

MPHT Review center: Solution and JAnswer key in MATH Page 48


18. N = (n -1)! = (10-1)! By repeated trial probability,
N = 362,8?a p = (nor)(p)I (q) ii_I
P = (5C2)(1/2)2{|/2)3
19. n = 13 teams P = 5/16
r = 2 teams per game
N = ncr = 13C2 = 78 29. Free throw average (p) = 0.65
Minimum no. of days for the tournament
q = 1 - p = 0.35
= 7816 = fl n = 3, r = 2, n - r = 1
By repeated trial probability,
20. n = 10, r = 6 P = (ncr)(p)r (q) n -r
N = ncr = 1 0C6 P = (3C2)(0.65)2(0.35)
N = 270 P = 0.444

21. n = 10, r = 3 points for triangle 30. Probability of getting red ball
N = nor = 10C3 P1 = 5/8
N = 720 Without returning the first red ball, the
probability of getting another red ball
222. ni = 9 men, ri = 7 men P2 = 4/7
n2 = 6 women, ri = 5 women P = (5/8)(4„)
N = (9C7)(6C5) P = 5/14
N = 216
31. If white ball is drawn from the second bag
23. ni = 7 men, ri = 3 men P1 = 3/8
n2 = 5 women, ri = 2 women lf this ball is placed in the first bag and
N = (7C3)(5C2) white ball is drawn
N = 350 P2 = 5'8
If black ball is drawri from the second bag
24. n = 5, r = 1, 2, 3, 4, or 5 P3 = 5/8
N = 5C1 + 5C2 + 5C3 + 5C4 + 5C5 If this ball is placed in the first bag and
N=31 white ball is drawn
P4 = 4/8
25. n = 42, r = 6 P = PIP2 + P3P4
N = 42C6 = 5,245,786 P = (3/8)(5/8) + (5/8)(4/8)
Probability of winning if you bet a ticket P = aE454
P = 1_/5.245,786
32. Even numbers (box A) = 2{ 4, 6, 8
26. For sum of 9, the possible outcomes are Even numbers (box 8) = 2, 4
(4, 5) (5, 4) (6, 3) and (3, 6) = 4 outcomes Probability of getting an even number
The total outcomes is 36 from box A,
p = 4es = 7ro P(A) = 1/2 (4/9) = 2/9
Probability of getting an even number
27. The positions of boys and girls where the from box a,
girls are always together P{B} = 1/2 (2/5) = 1/5
GGGBBB
BGGGBB P(T) = P(A) + P(B) = 2/9 + 1/5
BBGGGB p(T) = 19/45
a a a G G G 4ways Probability that the even numbered card
No. Of ways to arrange girls = 3! came from box A
No. Of ways to arrange boys = 3!
N = (3!)(3!)(4) = 144
p-#-#
Possible outcomes = 144 P = 7a/79
Total outcomes = 6! = 720
P = 144„20 = 1„ 33. p = 2/5 , q = 1 - p = 3/5
n = 8, r = 5, n - r = 3
28. Probabilfty of having a boy (p) = 1ra ` By repeated trial probability,
Probability of having a girl (q) = 1A2 p = (nor)(p)I (q) n-i
`n = 5, r = 2, n -r = 3
P = (8C5)(2/5)5(3/5)3
P -- 0 . 1 2 3 9

MPHT keview center: Solution and Answer-key in` MATH .,, Page 49
34. Enter Mode 3 2
XY 2. EnterMode2
10.2 {2 -3.i) {5 + 2.1) = 16 - 1_fl
2 0-28
3 0-18 3. i=VI;i2=-1i3=-i
4 0.23 i29 + pr + i = (j28)i + (j2O)i + i
5 0.11 = (ifm)i + (if®)i + i
Press AC, then Shift Stat Sum fxy = i + i + i = 3/
Zxy = ZZZ
4. Shift Mode 4 (Radian Mode)
35. Expected gain = Investment - Loss z = 4 + 5i
Expected gain = 40000(0.3) - 10000(0.7) pal (4, 5) =
Expected gain = ap r = 6.40. e = 0.896
ln (4 + 5i) = ln (6.40 i 0.896)
36. Probability of getting a card of club = ln (6.40) + jo.896
Pi = 13/52 = 1/4 ~-1±$56+io.896
For four cards with replacement,
P = {1/4)(1/4)(1/4)(1/4) = ±g§§
5. V=-(o-j)1/i
37. Ways of getting 4 aces out of 5 cards -(1£-Z)1'j
N = (4C4)(48C1 )

P- (4C4)(48C1) -(e-i:)1/,
(52C5) - e-t(/2
P--1/54145
-_0.2078
V_i
38. Probability Of getting head (p) = 1/2 6. EuterMode2
Probability of getting tail (q) = 7€ (1 + i)7= (1 + i)3 (1 + i)3 (1 + i)
n =5, r = 3 (1 + i)7 = ar;
By repeated trial probability,
P = (nor)(p)I (q} n_r 7. ShiftMode4 (Radian Mode)
P = (5C3)(1/2)3(1/2) 5-3 z = 3 + 4i
P = 5/16 = a.3t25 Pol (3' 4) =
r = 5, 6 = 0.927
39. Probability of getting head (p) = 1/2 ln (3 + 4i) = In (5 i 0.927)
Probability of getting tail (q) = 1/2 = In (5) + jo.927
n = 10, r = 7, 8, 9 or 10
--1_.60+io.927

By repeated trial probability, 8. 3x+4y+3yi+15-3i=0


P = I (ncr)(p)r (q) n-r 3x + 4y + (3y -3)i = -15 + jo
Consider the imaginary parts
P=±7[(Locx)(i)X(i)LT 3y-3=0 - y=1
3x+4y=-15 + 3x+4(1)=-15
P-rm x = - 6 . -?_?

40. Probability of getting head (p) = 1/2 9. Shift Mode 4 (Radian Mode)
Probability Of getting tail (q) = 1/a z = - 46 - 9i
n = 10, r = 4, n -r = 6 Pol (- 46' - 9) =
By repeated trial probability, r = 46.872, © = -2.948
P = (ncr)(p)r (q) n-r
(46 -9i)ire = (46.872 i _ 2.948)1ra
P = (10C4)(1/2)4(1/2)6 = 46.87218 £ (ire x -2.948)
P = 105/512 = 3.61 i -0`983
-- 2 - S i

10. Shift Mode 4 (Radian Mode)


z = 1+ i
1. z=3-4i,x=,y=-4 Pal (1, 1 ) =
Pol (3, - 4) a. r = 1.414, 9 = 0.785
a. r=5
b. 9=53.73o

MPHT Review Center.: Solution andLAnswer-key in MATH Page 50


For the first root, k = 1 15
( 1 + i)1/5 = (1.414 i (0.785 + 2TT(1 ))1/5 1 14 3 1
= 1 .4141/5 < (1/5 x 7.068) 1 5 -1 3
= 1.072 £ 1.4136 1 -2 2 -3
--0.168+1.059i 3 -4 -3 -4
Solve this matrix using cofactor,
For cofactor of let row, 1 St column
11. Enter Mc)de 2, Shift Mode 4
Enter Mode 6
sin(x + jy) = sin x cosh y + j cos x sinh y
Choose Mat A (1 ), 3 x 3 (1 )
sin(3 + 2j) = sin 3 cosh 2 + j cos 3 sinh 2
sin(3 + 2j) = a.537 - /3.59 -2 2 -3
A5-13
-4 -3 -4
12. Enter Mocle 2, Shift Mode 4 Shift, Matrix (4), Data (2)
cos( x + jy) = cos x cosh y -j sin x sinh y
cos(1 + 2j) = cos 1 cosh 2 -i sin 1 sinh 2 For cofactor of 2r'd row, 1st column
cOsci + 2i) = _2..ae_27 - i3.oEi_9_ Choose Mat a (2), 3 x 3 (1 )
1431
13. Enter Mode 6 8[ -2 2 -3
-4 -3 -4
Choose Mat A (1 ), 3 x 3 (1 )
Shift, Matrix (4), Data (2)
123
A[ -2 -1 -2
314 For cofactor of 3rd row, 1 ct column
AC Shift 4 7 Shift 4 3
det (Mat .A) = 5
:E5:Se=¥:at:§;3],,3x3tt]
14.
3 -2 4 2
Solve the partial determinant using Shift 4
2 1 -3 5
-4 0 -1 -2 = 1 (-1 )1+1 det (Mat A) + 1 {-1 )2+1 det (Mat
-5 3 2 4 8) + 1 (-1 )3+1 det (Mat C)
Solve this matrix using cofactor, =352
For cofactor of lst row, 1 St column Shift, Matrix (4), Data (2)
Enter Mode 6 For cofactor Of 4th row, 1St column
Choose Mat Choose Mat A (1 ), 3 x 3 (1 )
1-35 31
0 -1 -2 -13
2-3
Shift, Matrix (4), Data (2)
For cofactor of lst row, 2nd column X = 352 + 3(-1 )4+1 det (Mat A)
Choose Mat 8 (2), 3 x 3 (1) X=3ap
-35
8[2: -1-2 12
24 16. A = -10
Shift, Matrix (4), Data (2) E2 2-1
For cofactor of lst row, 3rd column BY transpose, interchange the elements
of rows and column§
), 3 x 3 (1)

17. Multiply the.matricest afthe right`side


Solve the partial determinant using Shift 4
= 3(-1 )1+1 dot (Mat A) + (-2)(-1 )1+2 det (Mat
2 _12] [2]
a) + (4)(-1)1+3 det (Mat C)
= - 275 M = z [Z{Z} _L2tf3,]
Shift, Matrix (4), Data (2) 1
For cofactor of lst row, 4{h column 2 LLL°o] = L55]
Choose Mat A (1 ), 3 x 3 (1 ) x -- 5 . v -±
21-3
-4 0 -1
-532 18.3[:7;I+2[::]=[f9£:3;I+I:§L:8]

x = -275 + (2)(-1 )1+4 det (Mat A)


x = -Lse 3[:7 :]+2[: :I-I:: :5:I

MPHT Review Center: SolutionandAnswerkeyin'MATH


Page 51
19. Mode 6
If s = 0'
Mat A (1 ), 3 x 3 (1 )
21
1=3A - A=±
3
A= 61 Then,
37
AC, Shift, Matrix (4), Mat A (3), x-1 F(s, -¥+#
-2/= Wf3/&5 1/€5.
A-1 0 -1/13 2/13 f(t,-L-1(¥)+L-1(#)
3/5 -11/65 -4/65 From L (eat) = 1 / (s -a),
£CO~-1©lrse-3t
20. For f(I) = tn
n!
L (tll) - 26-F(s)==
sn+1

L (t3) -
3!
f(t, - L-1 (=)
s3+1
L ([3) = 6's4 f(I,-L-1(±)+L-1(±)
21. For f(t) = cos at Since L (sin at:) = aF¥where a = 3
S
L(cosat) - and L (cosat) =
S
s2 + a2
S s2 + a2
L (cOs5t) -
s2 + 52 ftt,=4L-±trij+=L_±t±>
L (cos 5t3 ± a_ I {3 + _Zap
£®~-4±i2s!s±s3t+113sin3t
22. For f(t) = sin t and n = 1
27. Resolve F(s) into partial fractions
L[tnf(I,I-(-1,n#(L[f(t,,,
F(s) -
4sA8
(s + 2)(s -6)
=in+in
L[tsint]=(-1)[±(L{sint)) Multiply both sides by (s + 2)(s - 6)
4§ = A (s -6) + B(a + 2)
But, L [sin at] =a _ where a = 1 If s = - 2
4(-2) = A(- 2 - 6) -A=1
L [tsint] --
`:='-i`.-,; If s = 6,
4(6) = 8(6 + 2)
L [tsin t] -- (s2 + 1)(0) -1 (2S) +8=3
(s2 + 1)2 Then,
L [t sin t] = 2s /./s2 +_fl2
F(s) = 13
FH + *
23. For f(t) = cos 2t f(I:,-L-1(±)+L-1(=)
S
L(cosat) - From L (eat) = 1 / (s - a),
s2 + a2
L (cos2t) = ± = ffi Ss £® - g=2t_+_3±
By First Shifting theorem, replace s by s - 28. By reverse of First Shifting theorem,
3, since L (e3t) = 1 / (s - 3)
c L(e3tcoszt)=F(S-a) F(s) - (s + 2) (s + 2)
(sz + 4s + 20) (s + 2)2 + 42
L Ce3t cos ZO = {s - 3}1 ifts - 3}2 _ +_44 Then a = ~ 2 for eat and k = 4 for cos kt
The inverse Laplace transform
24. F(s) = 3 / (s2 -16) f® = 9icosL4±
ftt:,=L_Lt=j==L-Ltifej
a
SinceL(sinhat)=rfewherea=4
£®=324:s_inn__(_4ff}_
1. Given A=
G?) subtract A from the
25. Resolve F(s) into partial fractions elements of .the principal diagonal
lAB 0 = A - ^1
Fts]=giv=T+as 2-A
0=
Multiply both sides by s(s + 3) (2;
1 = A (s + 3) + Bs 0=(2-A)(1-A)-3(4)
lf s = - 3 ^qu=5and^®__-±
1=B(-3) - a--:

MPHT Review Center: Solution and Answer key in MATH


Page 52
2( 5-^3 =0 17. f(x) = x + (1/2) x2 + (1/3)x3 + (1/4)x4 + .
02-A ) By inspection,
(5-A)(2-A) -3(o)=o
^an_and^.__--± f(x,=£=
n-sO
1-^2
3-( 18. f(x)=£=-.£
32-A
(1-A)(2-^)-3(2)=0 f(x) = er
^ =4,-1
If ^ = -1 19. i(x) = x2ex
';`i x3 f(x,=x2(I:==o:,
x4 )-(1_!„ 2-2-1))-(3 :)
i(x) = I:£so

20. f{0)=coS(0)=1
4. Since 3 x 3 matrix, there will be ±Lse£ Of
f'(0)= - sin(0)=O
eigenvectors 3 x 1 matrix
f„(0)=-cos(0)= - 1
e. F`eferto#1
Ft.i:{8o,,==8Insttoo,,____0+
Eigenvalues ^i=5and^2=-2
2-5 3€)-(: .34) coefficientofx4=fe=J
lf ^1 =5, 4! 4!
41€
Coefficient of X4=
lf ^2=- 2, ZZ
21. fi{x) = ln x
Eigenvectors:
I(x)=;
Modal Matrix:
i"(x) =

10. f(t) = t-1'2


For the coefficient of (x ~ 1 )2 at a = 1
I"===-1
£[f(t,]="t-ira,=%=
coenideutamx.„4__-i.__i
Where (i) !=r(-:+1)
22` For the coefficient Of the (x -a)4 at a = 3
€)!=r(:)if
Then, I(t-1/2)=g f;)(:)==ff((3X,,=2

•(("2)-k coeffictientof€X-3)4~-+#=%~-jz2
coefficient of (x . 3)4= 7
1 1 . r(8) = r(7+1) = 7! 73
rc®= 5040
23. For f(x)=vi , where a = 16
f(16) = 4
12. r(io) =r(9+1)=9! f,(x,-!x-i
= 9 r(9)
± a r(eti) = 9 (a)I-(a) p(1e,-ix-i
Then r{10) is not equal to ton(9) f"(x)-:(-:)
f`(16)=±(-±)(16)-3=-i
Z56

13. cOs© = gap The power series.will be


2
1
e j2oTTt + a -j2otrf
6CoS20TTt =
f(x)=4+:(x-16)-a(x-16)2
6Co§20TTt = 3e/20ut+3e-/2out
t¢xfe4+18¢*i6j-±26*iGf
14. For the term ejl0Trt+e-jl0Trt,
At t=1. (set into radian mode) 26. Assume x = -1, n = 1
ejl oTrt+e-jloTrt = 2
e JIon + e `JIOTrt By trial and error, (assume 100 for infinity)
Choose A
= 2CoS10TTt
= 2co§(10iT(1 )) f(x,=3+
=2'- i(=[1-cosrm])sin(¥)
n=1

MPHT Review Center: Solution and `Answer key in'' MATH Page 53
f("=:+£(=[1-cosnq])sin(=) MODE 5 - 1
ab
n-1
f(-1 ) = 0
1-2
23
Assume x = 1, n = 1
8=18yearsold;E=±g±d
3- A=2:rty
f(1)=:+±(±[1-cosnlt])sin(=)
A = 2x(12 -x2) = 24x _ 2x3
n=1 dA=24-6x ,
f(1 ) = 3
0 = 24 - 6x
Then, the function is
x=4
A = 2(4)(12 -42) I -32
f(x,=:+ A = 32Jsa--
i(±[1-cosn"])sin(=)
n=1
4. Let x=amountof3%iodinesolution
27. n(n -1)(n -2)(n -3) ... (3)(2)(1) = a!J y = amciunt of 20% jodjne Solution
The amount of the solutions
28. Fundamental frequency = 20TT x+y= 85 -Equation 1
Ou =2Trf The concentrations of the solutions
2OTT =2Trf 3x + 20y = 85(19) -Equation 2
f_-ap
MODE 5 - 1
34. The terms are perfect square, then for the abc
1 Oth term, 1185
aio=(10)2=flo
S=5;y2=°80 85(19)
Harold used 5 ounces with 30/o iodine
solution

5. Let x = number offirst type seeds ($1)


y = number of second type.seeds ($1.26)
10-x y = x + 50
($1 )(x) + ($1.26)(y) = $402
x + 1.26 {x + 50) I 402
20 y 20 x--anDacke4§
6. Let x=smallerangle
X
90 - x = larger angle
90 - x = 2x + 1 5
2 0> X=3±
Let 2x = length of rectangle, y = width of 7 t83yx:;t::3;;4e:Pff6x„
rectangle
lf xo = 10, ho = 20,

yo=jiirfu,~_ioI€3
8. Forthe area ofrectangle
By ratio and proportion of similar A = length (width)
triangles, A = (2X)(4X + 5)
-E=
y
10-x
r=-±
yo
xo
-
10 JB
10
A=892±+1g_2£_

9. Let x and y be the dimensions c>f the


rectangle
X=¥&}`=°2-xX()+3)(|o_x)
A=yry
A = 2Vg (10x -x2)
Differentiate A with respect to x 1 00 = xy
y = 1 00/x
£=2t6(10-2x)
0 = 2Vg (10 -2x) For the perimeter
x=5 P = 2 (x + y)
P = 2 (x + 1 00/x)
A=38;)!V3Hio_5)

2. B=2E-2 ggff=r:(ntti:t:oPo/¥2t)hrespecttox
B-2E=_2 + Equation 1 0 = 2(x2 - 100)
2B+3E=66 + Equation 2 x = 10, y =.10
Then the minimum possible-perimeter
P = 2 ( 1 0 + 1 0 ) = io

MPHT Review Center: Solution.-and.AmswerkeyinLMATH


Page 54
10. Let x=diagonal
15. Let x = increased dimension of the garden
X - 5 = length
X - 7 = width _=i]_i_:I_:i_:i:__:_i
A = length (width)
195 = (x ~ 5)(x - 7)
X = 2Qm

1 1 , Let L I 2W + 2 24+2x
A=LW
3280 = LW
3280 = W ( 2W + 2 )
W -- 4e a 20+2x.
12. Let C be the point on the other side of the Anew = Aorig + 141
river directly opposite A. Let D be the
point between C and 8 that Frank should (20 + 2x) (24 + 2x) = (20) (24) + 141
swim to; he will run the rest of the way to x = 1.5 ft.
8. Let x be the distance from C to D.
The new length of the garden
Lnew = 24 + 2X = 24 +2(1.5) = 2ZL#

16. Refer to #1 for solution


^-559±
17. In ABCD, zA = £C and £8 = £D, then zA
T = tswIM + tRUN
and £D are supplementary

I-±+* ch + £D = 1 80
3x + 10 + 2x + 30 = 180
x=28
I-=+i= zA = 3(28) + 10 = 9±
Differentiate T with respect to x 18. For ini{jal volume; Vi = TTr2h
dT For final volume, let r2 = 2r
Th-== :(1+x2,=(2x',+(i) V2 = TT (2r)2(3h) = 12 iTr2h
o =¥(1 + x2)=
V2 = 12Vi
x = 0.75 miles The volume .is multi lied b 12

T- J- - 3 - o.75
35
19. A = L x W
dA
T = 0.87 hr ( 60 mins / hr ) E -L ¥+ w#
I--anLe
After i = 3 see.
1 3. Let x = width of the walkway W = 20 + 3t = 20 + 3(3) = 29 in
= 20 -2t = 20 -2(5) -= 14 in

(14in)(3=)+(29in)(-2±)
24+2x - 16 in2/see
|6in2/sec=deereas_inng|
i_:i__:_T
35+2x
For the area of the garden surrounded by
walkway
( 35 + 2x ) ( 24 + 2x ) = 1 530 18
X _- a n

14. .Let Bi = time for Ben to work alone `and 82
= time for Bill to.work alc]ne
111
15x-
E:+i;=F Letx=.length`ofthe.shadow.Of the.tree
111 By similar-triangles,
18.6
F+in=a __=-
x+15 x
Bi--rmhoap x = ZEE
MPHT::Review.Center:
Page 55
21. The area of the grid square is 2 in x 2 in =
4 sq. in. •o-¥(5)2g
dhlclt--0.032ft/min

27. Let N = no. of combinations of 5 card


hands from the deck
N ''~ 52C5 = 2±
28. If the plane's two engines operate
independently,
P = Pip2 = (1/100)(1/100)
P--OQan
2!£.y2(2+y2-~_2!5
Differentiate the equation
22. Let Q = probability that every 2 people
have different birthdays :x¥¥++y2#:o=O
For 30 people,
336 At x = 3, y = 4 and dx/dt = 2
Q = 3#x3%x¥x...x 535
365P30 3(2,+4g=o
3653o dy7dt = #cLrfe
Q - 0.29
Let P = probability that at least 2 people 30. If the sum of the dots of pair Of dice js 5,
have the same birthday Possible Ouitcomes: (1,4) (2,3) (3,2) (4,1 )
P = 1 ~ Q = 1 ~ 0.29 = 9izz Possible Outcomes = 4
Total Outcomes: 6(6) = 36
F.or problems 23, 24 and 25 P = 4/36 = JZ4!
23. Enter MODE 8 and use in = 3
For problems 31, 32 and 33
AB = a - A 31. By conditional probability,
AB=<4-2 2-2 1-2>~VCTA P (A and a) = P(A) P (B/A)
P (A and 8) = (1/2)(1/3)
AC = C - A P{AandB)=fl6
AC=<2-2 3-2 1-2>~-VCTB
32. P (A or a) = P (A) + P (a) - P (A and 8)
Using SHIFT 5 7 to solve cos e by Dot 8) = 1/2 + 1# -1/6
Product ± Fill
Vct A Dot Vet 8
cos© 33. The probability of A under a, P(A/B)
Abs(Vct A)Abs(Vct 8)
cose-_1/JI0 P (A and 8) = P(B) P (A/B)
1/6 = (1/2) P (AIB)
24. Area = 0.5 (AB)(AC) P (AIB) = 4£
Under Mode 8, solve the area by Shift 5
34. The boundaries involved are yi = a, yu = x2
A = 0.5 Abs ( Vat A x Vet a ) Using Ring method,
A=i5 V = nJ.fyu2 -y{2) dx
V = " /az((x2)2 - 0) dx
z5. The vector that is perpendicular to the V = a r5
plane is the cross product Of the two
vectors on the plane 35. There are 6 ways to roll a sum of 7
By Shift 5 under Mode 8,
(1,6) (2,5) (3,4) (4,3) (5,2) (6,1)
AB x AC = Vct A x Vct a P (sum of 7) = 6/36 = 1/6
A!B x Ale -~ < 1 2> There are 6 favorable outcomes with 36
equally likely outcomes, .so 30 are
26. Given: dv/dt = 10 ft3/min unfavorable. The odds in favor of a sum
Find dh/dt at h = 5 ft of7are6©orlys
Since r = 2h
V=gr2h==(2h)2h 36. Enter Mode 2 (Complex), Shift Mode 4
(Rac')
v=¥h3 Convert 3 + 4i into-polar form
Enter 3 + 4i then Shift 2 3
= 5 £ O,927r
The derivative of V with respect to t
ln (r f e) = ln (re JB) = In r + je
¥-¥h2# ln (3 + 4i)'= ln (5) + jo.927
in(3+.4i)=£frt_..+/:OLff

MPHT.Review Center:.Solution:.and`Answer key:-imMATH


Page 56
37. Solve the values of x at y = 0
4x - xz - 3 = 0
Enter Mode 5 3
abc
-1 4 -3
X1 = 3, X2 = 1

For the enclosed area


A= I (yu -y£) dx
A=xy
A= Jt3(4x -x2 -3) dx A = X ( 50 - x ) = 50 - x2
A = 4/3 Differentiate A with respect to x
dA = 50 - 2x
40. Let i + i2 + i3 + i4 = i - 1 _ i + 1 = 0 Set dA = 0,then x = 25
Then, Then the maximum area for her garden
i + i2 + i3 + . . . + i23 = i21 + i22 + i23 A = 25 (50 - 25)
i + i2 + i3 + . . . + j23 = i + i2 + i3 A=625s .ft.
i + i2 + i3 + . .. + i23 = i _ 1 -i = -7.
45. A = J' y dx
41. Solve for the x if y = 0,
A = /o2(x3- 3x2 + 2x + 1) dx
0 = x2 - 5x + 6
Enter Mode 5 3
A--2
ab
1-5
46. For the avi6rage of 93% for 4 subjects,
Total scores Of 4 subjects = (93)(4) =, 372
X1 = 3, X2 = 2

Missing grade = 372 - (85 + 92 + 95)


The area bounded from x = 0 to x = 2
Missinggrade=flo
Ai = / y dx = /o2(x2 - 5x + 6) dx
A1 = 14/3 47. Given: 320 cans, 40 cans per box
The area bounded from x = 2 to x = 3 No. of boxes = 320 cans / 40
No. of boxes = 8
pe = I y dr = f23 cx2 - 5x + 6) dr
A2 = - 1 /6
The area bounded from x = 3 to X = 4 48. Solve x at y = 0 (x - axis)
x - x2 = 0
A3=/ydx=J34(x2-5x+6)dx
x (1 - x) = 0
A3 = 5/6 xl = 0, x2 = 1

The total area Using Disk method


A = |Ai| + |A2| + |A3|
V = " /fyz) dx
A = 14;3 + 1;6 + 5/6 = an
V -- Tt Io. Cx - x2)2 dx
42. Given:ai=3 an=136 V=zzZIOLO

Sn=:(ai+an) 49. From 10 AM (Fri) to 10 AM (Sat)


1390=:(3+136) ti = 24 hours
From 10 AM (Sat) to 6 PM (Sat)
n=20 t2 = 8 hours
T = ti + t2 = 24 + 8 = 32 Aours
Enter Mode 3 - 2
50. If she pledged 0.50/mile. then

136 I3!:i::g::i:fi#es,,So.5o,in"e,
Solve for a2 and a3 where 9 = Shift 1 5 5
a2 = 29 = 1 0
a3 = 3y = 1 7
Triefirst3termsare±LJOandL±Z

43. Based on the table,


i(3) = 2
g(f(3)) = g(2) = £
44. Let x and y = dimensions of the fencing
The perimeter Of the garden
2x + 2y = 1 00
x + y = 50
y = 50 - x

MPHT Review Center: Solution and..Answerkey.in..MATH


Page 57
ANSWER KEYS 24. a. -8 hours
25. A. 21
EXERCISE NO.1: AIgebra -Part 1 26. A. 342.85 mph
27. 8. 73.33 mph
1. A.4 28. 8. 24 minutes
2. C.2 29. C. 43 7/11 minutes
3. A.4,-5 30. 8.12:16:21 am
4. D-4,3
5. A.16 EXERCISE tNO. 3: Trigonometry
6. A-4
7. A.2 1. A.-cosA
8-A.4,-7 2. A.3/5
9. C.1 3. a.-1/7
10. D. 8 4. A.5/13
11. D. 5 5` A.120aor240°
12. a-12 6. A.9
13. C. 2x5 7. B.358.52m
14. 8. |2ox7y3 8. D.196km
15. A. -1365x4yll 9. A.3.732km
16. D. 43750x8 10. C.14.78 meters
17. A. -192456 11. 8. 31
18. A.1 12. C. 22.87 in
19. D.19684 13. D. 90 in
20. C.110 14. A.10.2 meters
21. 8. 28 15. D. Sin 4y
22. D. -28 16. C 2/3
23. A. 2 17. C. 74.16 in
24. A. 42 18. D` 300
25. A . x3 - 5x2 + 2x + 8 = 0 19. C. |35o
26. A. 50 20. 8. 800mils
27. C.120 21. C. 74o2o`
28. D. 9 22. a. 4 P.M.
29. A. 230 23. C. 6 A.M.
30. D. 3000 24. 8.158°18`43"
.31. C. irrational 25. A. 56,476,062 km2
32. 8. transistive 26. C. cos(- ©) = cos(©)
33. D. imaginary 27. A.I,Ill
34. 8. real,unequal;rational 28. a. IV
35. a. extremes 29. C. latitude
30. A. angle of depression
EXEF{CISE.` NO. 2: Atgebra` - Part 2 31. a, quadranfa[
32. A. 2sinocos©
1. C.120 33. C. reflex angle.
2. C.n2 34. 8. The sum of three angles is 18o°
3. C.31 35. A. 45 degrees
4. C.128 36. A. 3094
5. C.0-02 37. a. linear pair
6. a-8 38. 8. Medians
7. C.62730 39. a. bearing
8. 8.50 40. A. Th/3
9. A.`5
10. A. 5
11. C. 204.8 in
12. D. 7, -7/6
EXERCISE NO. 4: Planeiand`Solid
13. A. 29524 Geometry
14. C. 3/4
15. D.1/22 concave
16. A. 0.033 360
17. D.15
18. a. 20
19. C. 24
20. C. 2
21. 8. 20%
22. C. 2.18
23. a. 4/3 hours 259.81
120.71 cm2

MPHT Review Center: Solution. and.`Answer key in MATH, Page 58


11. A. 206.71
12. C. 0-5
12. 8. 40.428 sq. cm
13. D.. Downward
13. a. 432
14. C. 78.54 sq. in
14. A. 5x2 + 4x -10y + 3 = 0
15. a. 5
15. D. 96 sq. cm
16. D. 96 in
16` C. 31.2 sq. in
17. 8-(2, 3)
17. D. 6.382 in
18. a. 4
18. A.1.71 cm
19. D.1,5
19. C. 5 cm
20. C. 4x2 + 9y2 - 8x - 36y + 4 = 0
20. A. 500 in
21. a. 0.92
21. C. 4.5
22. a.15TT
22. a. 4
23. D. 3-6
23. C. 32.23 cm
24. C- 4
24. D. TTr3/3
25. C-(-1, -1)
25. a. |8m3
26. C.12
26. A. 23%
27. A.1.8
27. C. 45 pi
28. a- 2
28. 8. 2,670.35 cm2
29. a. 4x -3y -20 = 0
29. a. 243.78 cm2
30. a.1.41
30. A. 216.5
31. a. 5.33
EXERCISE NO. 5: Analytic Geometry _ 1 EXERCISE NO. 7: Differential calculus

26- 8. 11 1. B-%
27. A.15.65 2. a.3
28. a.11 3. a.2/5
29. D. 5.38 4-B. Infinity
30. A.11/3 5. a.2
31. C. (13, -20) x2+ 2x
6.A
32. D. (-1,1) Eiiiii
33. C. 6
34-A.1,2 iix(:::i:2eT/%2Sint¥2}
I(x2:2+2:}1r2
35. C. 38
10. A. 2x - y = 0
36. D. 25
11. A.19.8 in/s
37. a. 20
12. a. 4.94
38. D. 2x -y = 3
39. A. No!
13. C. -2 ± JZ
40. 8. y -x + 4 = a 14. A. ( -2. 28) , (2. d) , & (0,12)
15. C. 2x + y = 9
41. D. 2x-y + 11 = 0
42. A. 4x + 6y -29 = 0 16. C. 25 & 25
17. A.15.59 cm2
43. D. 8x + 4y - 4 = 0
18. A.12 in
44. D. 3
19. a.100
45. C. 4
46. A. 2.53
20. C. 22.24 cm, 44.5 cm
47. B. 4.39 deg. 21, 8. 0.95fflmin
22. D. 12 kph
48. C. x + 3y -12 = 0
23. a.1.33 in
49. A. 9x + 33y = 154
24. A. 6.28 ft3/min
50. 8.10.14
25. A. 0.012732 inch/see
51. 8. (10, 233.23°)
26. C. -0.001 amp/see
52. D. 5x2 + 9y2 + 12x -9 = 0
27. C` Point of Inflection
28. 8. Minima
29. D. Zero
EXERCISE NO. 6: Analytic Geometry _ 2 30. B. f'{a) = 0
EXERCISE..NO..8::.Integral.;Calculus
a
Ola 1. C.3TI/16
et 2. A.O.134
3. A.0.0184
4. C.17.5
6x + 10y +18 = 0
5. a.9
units
6. C.34X(|n81)+C
6x - 1 8y - 1 0 = 0
1 6x + 2y + 52 = 0 7. D.2tanx+cosx+C

MPHT Review Center: SolutiomandAnswerkeyin'MATH


Page 59
4. C.(0,0,-2)
8 A €(X3+3)3/2+c 5. A.1o.3o5o
6-a-(4'-6=

9. D ±(2y2+1)"2(3y2_1)+c 7. C-=i-=j
8. A.-3,4/3
|o. c. cos(1/x)+c 9. D.(a,-£,(b,gi-=j
11. D. ey4 +c 10. A. 32
12. a. 2 11. a. J55
13. A. 9/2 12. D. <13, -7, 2>
14. C. 21.oe 13. C. <~3, 3,10>
15. a. 2Tr 14. A. <-12, 21, €>, <20, -35,10>
16. 8. 44.57 sq. units 15. a. ±i+±j + ±k
17. a.1.42
16. A_ 17.83
1 8. A. 2TT, 4lT
17. a. 2 cm
19. C.1.125, 3.6
18. 8` 138
20. D. 0.582
19. C. 2i + 7j -5k
21-a-1.5
20. A.15.0
22. A. 0.571
21. a. 3
23` c. 4rf 22. a. 24
24. A. 0.095
23. C.13.6
25. a. 50.265 cu. units
24. C. 74°|2'
26. A. 28.27 cu. units
25. D. -18i + 3j -15k
2:I . A. 2:2f3
26. A. 3`73
28. C. 26.81 cu. units
27. A. -1.143i + 4.573j + 2.287k
29. D. 48TT2
28. A. 3`39
. 30. a. 972.16
29. A. 3i -j + 5k
31. D. 624
30. A. 5.92
31. a. -2.19
EXERCISE NO. 9: Differentia.I Equation 32. A. < -.828i -1.655j -1.241k>
33. a. 84°2o'
1. a.2
34. A. 3
2. C.3
35. D.15
3. D-2,4
36. a.13.7
4. A. (x-2y)dx-xdy=0 37. D. (ra)i -(yz)i -(x2 + y)k
5. A.y-y-&ry=0
6. D.ydx-(x+1)dy=0 EXEF2C[SE NO.11 : Sfatistjes.`€&:Probability
7- 8- yy" + (y')^2 + 1 = 0
8. . D. tan-1 x.+ tan-1y = C 1-8.59
9. A.y=sqrt(5x-1) .
2. C.37.75
10. C. x^3 = c(9x^2 + y^2) 3. A.40.78
1 1 . A. (x + y)dx + (x - y)dy = 0 4. a.52.00
12. A. xR:R:y + y -_ C 5. 8.9.49
13. 8. yy" -2ycos x = sin x 6-C.1.7
14. 8. e3x 7-a.13
1 5. A. y = 8e3X _ 2 a. C.95
16. C. y-1 = ce(x^2)Z2 + 1
9. A.56
17.A.y=e[.5X(c[cos¥x+c2sin¥x) 10. A. 360
1 8. A. y" + 2y" + 9y' + 18y = 0 11. 8. 240
19. 8. y = ex(ci + xc2 + x2c3) 12` C. 72
20. C. 694 13. A.144
21. 8. 39.6`min 14. A. 360
22. A. 2.5 sees 15. A. 64
23. C. 0.338 min 16. C. 34650
24. A. 0.5x^2 + y^2 = k 17. a. 2520
18. A. 362,880
EXERCISE: NO.10:'Plane.and:Spage 19. C.13
Vectors .-'`. 20. A. 210
21-A.120
22. a. 216
23. A. 350
24. C. 31
25. C. 1/5245786

MPHT Review Center: Solution and.Answer key in MATH


Page 60
26. A. 1/9
7. B. Spectral matrix
2:I . A. 1 15
8. B. Spectral matrix
28. A. 5/16
29. C. 0.444 ?a. 8: ?(i:%u,,ar
30. C. 5/14 11. a. 5040
31. A. 35/64 12. c. tor(9)
32. a.10/19 13. a. 3e020ut) + 3e(-j2oTrt)
33. C.12.38% 14. C. 2
34. a. 2.77 15. A. Odd function
35- A. 5000 16. D. 36oo
36. C.1/256 17. C-Z:a-1¥
37. A.i/54145 18. A. ex
38. A. 0.3125
19. C-I:;=o¥
39. 8` 11/64
40. A.105/512 20. D.1/24
21. C. -1/2
22. C.1/12
EXERCISE NO.12: Advanced-+Engineering
23. D. 4 + 1/8 (x -16) -1/512 (x -16)2
NIathematics - 1 24. 8` Convergent
25. A. Divergent
1. A.5,53.|3o 26.A.:+I:=L(=[1-cosrm])sin(=)
2. D.16~11i
27. C. n!
3. A-3i
28. C.10
4. A.1.86+0.90i 29. C. Solidus
5. a.0.2078 30. A. Euler
6. B.8-8i 31. A. Surd
7. B.1`61+C)`93i 32. 8. on the x axis
8. A.-6.33 33. 8. an even function
9. B.2-3i 34. D.100
10. C. 0.168 + 1.06i 35. A. Euler
11. a. 0.531 -3.59j
12. C. 2.0327 -3.0519i MostValuable,Assessment(MVA)Testln
13. a. 5 NIatfiematics - 1
14. A. -385
15. a. 373 1.C2.a 21.C 41.C
22.a 42.A
3.a 23.A 43.a44.8
4.A 24.C
5.A 25.C 458
2 6.A 26-C 46.a
18. A. 7.A 27-a 47.C
2845 8.D 28.a 48.a
-2|5 rf3/es Lies 9.D 29.C 49.D
19.8. 0 -1/13 2/13 10.D 30.A 50.A
3/5 -T|/CJFi -4/6F] 11.A 31.C
20. a. 6/s4 12.A 32.a
21. 8. s/(s2 + 25) 13.a 33.D
22. 2s/(s2 + 1 )2 14.D 34-D
23. A. (s - 3)/((s - 3)2 + 4) 15.D 35.a
24. a. 8/4 sinh 4t 16.C 36,A
25. A.1/3 -1/3e-3t 17.D 37.D
26. 8. 4cos`3t + 1/3 sin 3t 18.A 38-A
T] . A. a-21 + raf ±idit 19.a 39.D
28. a. e-2t cos 4t 20.a 40.C

EXERCISE NO.13: Advanced Engineering


Mathematics - 2

Modal matrix
1-3

MPHT.Review center.:.. Soluti on:.and..Answer.key in..MATH Page 61

You might also like